SlideShare a Scribd company logo
Fluids
and
Electrolytes
BY: ROMMEL LUIS C. ISRAEL III
1
Fluids
HOW IMPORTANT IS WATER?
• Between 50% and 60% of the human body by weight is water
• Water provides a medium for transporting nutrients to cells and
wastes from cells and for transporting substances such as
hormones, enzymes, blood platelets, and red and white blood cells
• Water facilitates cellular metabolism and proper cellular chemical
functioning
• Water acts as a solvent for electrolytes and nonelectrolytes
• Helps maintain normal body temperature
• Facilitates digestion and promotes elimination
• Acts as a tissue lubricant
VARIATIONS IN FLUID CONTENT
BODY FAT
Because fat cells contain little water and lean
tissue is rich in water, the more obese the
person, the smaller the percentage of total
body water compared with body weight.
This is also true between sexes because
females tend to have proportionally more
body fat than males.
There is also an increase in fat cells in older
people
VARIATIONS IN FLUID CONTENT
AGE
AVENUES BY
WHICH
WATER
ENTERS AND
LEAVES THE
BODY
↓Blood
volume
or ↓BP
Volume receptor
Atria and great veins
Hypothalamus
↓
Posterior
pituitary gland
Osmoreceptors in
hypothalamus
↑Osmolarity
↑ADH Kidney
tubules
↑H2O
reabsorption
↑vascular
volume and
↓osmolarity
Narcotics, Stress,
Anesthetic agents, Heat,
Nicotine, Antineoplastic
agents, Surgery
ANTIDIURETIC HORMONE REGULATION MECHANISMS
Juxtaglomerular
cells-kidney
↓Serum Sodium
↓Blood volume
Angiotensin I
Kidney tubules
Angiotensin II
Adrenal Cortex
↑Sodium
resorption
(H2O resorbed
with sodium); ↑
Blood volume
Angiotensinogen in
plasma
RENIN
Angiotensin-
converting
enzyme
ALDOSTERONE
Intestine, sweat
glands, Salivary
glands
Via vasoconstriction of arterial smooth muscle
ALDOSTERONE-RENIN-ANGIOTENSIN SYSTEM
Fluid Types
• Fluids in the body
generally aren’t
found in pure
forms
• Isotonic,
hypotonic, and
hypertonic types
• Defined in terms
of the amount of
solute or dissolve
substances in the
solution
• Balancing these
fluids involves the
shifting of fluid
not the solute
involved
Isotonic
Solutions
• No net fluid shifts
occur between
isotonic solutions
because the
solution are equally
concentrated
• Ex. NSS or 0.9SS
Hypotonic
Solutions
• Has a lower solute
concentration than
another solution
• Fluid from the
hypotonic solution
would shift into the
second solution until
the two solutions had
equal concentrations
• Ex. Half normal or
0.45%SS
Hypertonic
Solutions
•
•
•
Fluid
Movements
• Fluids and solutes constantly
move within the body, which
allows the body to maintain
homeostasis
• Fluids along with nutrients and
waste products constantly shift
within the body’s compartments
from the cell to the interstitial
spaces, to the blood vessels
and back again
Fluid
Moveme
nts
•
•
•
•
•
•
Assessment
•
•
•
•
•
•
•
•
•
•
•
•
•
•
•
•
•
•
•
•
•
•
•
LABORATORY TESTS FOR
EVALUATING FLUID STATUS
•
•
•
•
•
•
Diagnosis
•
•
•
•
FLUID BALANCE
• The desirable amount of fluid intake and loss in adults ranges from
1500 to 3500 mL each 24 hours. Ave= 2500 mL
• Normally INTAKE = OUTPUT
FLUID IMBALANCE
• Changes in ECF volume = alterations in sodium balance
• Change in sodium/water ratio = either hypoosmolarity or
hyperosmolarity
• Fluid excess or deficit = loss of fluid balance
• As with all clinical problems, the same pathophysiologic change is
not of equal significance to all people
• For example, consider two persons who have the same viral
syndrome with associated nausea and vomiting
FLUID DEFICIT/HYPOVOLEMIA
• May occur as a result of:
– Reduced fluid intake
– Loss of body fluids
– Sequestration (compartmentalizing) of body fluids
Pathophysiology and Clinical Manifestations
DECREASED FLUID VOLUME
Stimulation of thirst
center in hypothalamus
Person complains of thirst
↑ ADH Secretion
↑ Water resorption
↓ Urine Output
Renin-Angiotensin-
Aldosterone System
Activation
↑ Sodium and
Water Resorption
↑ Urine specific gravity
Pathophysiology and Clinical Manifestations
UNTREATED FLUID VOLUME DEFICIT
Depletion of fluids available
↑ BODY TEMPERATURE
Dry mucous membranes
Difficulty with speech
Cells become unable to continue
providing water to replace ECF
losses
Signs of circulatory collapse
↓ blood pressure
↑ heart rate
↑ respiratory rate
Restlessness and Apprehension
Hypovolemia
• Nursing Intervention
• Monitor fluid intake and output
• Checked daily weight (a 1lb(0.45kg) weight loss equals a 500 ml fluid loss)
• Monitor hemodynamic values such as CVP
• Monitor results of laboratory studies
• Assess level of consciousness
• Administer and monitor I.V. fluids
• Apply and adjust oxygen therapy as ordered
• If patient is bleeding, apply direct continuous pressure to the area and elevate it
if possible
• Assess skin turgor
• Assess oral mucous membranes
• Turn the patient at least every 2 hours to prevent skin breakdown
• Encourage oral fluids
Hypovolemia
•
•
•
•
•
•
•
•
•
•
Collaborative Care Management
Identification of vulnerable patients and risk factors:
* Compromised mental state
* Physical limitations
* Disease states
* Limited access to adequate food and fluids
Development of a plan of care
Family members should
be educated about the
importance of fluid and
nutrition intake
Collaboration with the
nurse, patient, family
members, and other
health care providers
for continued
assessment and
treatment of problems
Ongoing assessment and
detailed action plan of
fluid and serum
electrolyte balance.
Factors such as
medications (particularly
diuretics),
hyperventilation, fever,
burns, diarrhea, and
diabetes with
appropriate referral
Collaborative
Care Key
Points
•
•
•
•
•
•
•
•
•
Fluid
Replacement
Therapy
•
•
•
•
•
•
•
•
Fluid Replacement Therapy
•
•
•
•
•
•
Fluid Replacement Therapy
•
•
•
•
Fluid Replacement Therapy
•
•
•
•
•
•
Composition of Different Intravenous Solution
IVF Dextrose
(g/L)
Na
(meq/L)
Cl
(meq/L)
K
(meq/L)
Lactate
(meq/L)
D5 0.9% NaCl 50 154 154
D5 0.15% NaCl 50 25 25
D5 0.3% NaCl 50 51 51
D5 0.45% NaCl 50 77 77
D5 IMB 50 25 22 20 23
LRS 0 130 109 4 28
NSS 0 154 154
D5LRS 50 130 109 4 28
31
Fluid Replacement Therapy
ISOTONIC SOLUTION
Facts Examples Uses
-same osmolality as plasma (app.
275 to 295 mOsm/kg)
-vascular space osmolality not
altered by infusion
-expand intracellular and
extracellular space equally;
degree of expansion correlates
with amount of fluid infused
-no solution-related shifting
between ICF and ECF spaces
-cells neither shrink nor swell with
fluid movement
Dextrose 5% in water,
Normal Saline Solution,
Lactated Ringers
Solution
-Fluid loss and dehydration
-Hypernatremia
-Blood transfusion, fluid
challenges, resuscitation,
shock, metabolic alkalosis,
hypercalcemia,
hyponatremia
-Acute blood loss, burns,
dehydration, hypovolemia
32
Fluid Replacement Therapy
HYPOTONIC SOLUTION
33
Fluid Replacement Therapy
HYPERTONIC SOLUTION
34
FLUID EXCESS/HYPERVOLEMIA
Psychiatric
Disorders, SIADH,
Certain head injuries
Dietary Sodium
Indiscretion
Renal and endocrine
disturbances,
malignancies, adenomas
Overhydration
Excessive Sodium
Intake
Failure of renal or
hormonal regulatory
functions
FLUID VOLUME EXCESS/HYPERVOLEMIA
• Since ECF becomes
hypoosmolar, fluid moves into
the cells to equalize the
concentration on both sides of
the cell membrane
• Thus there, is an increase in
intracellular fluid
• The brain cells are particularly
sensitive to the increase of
intracellular water, the most
common signs of hypoosmolar
overhydration are changes in
mental status. Confusion, ataxia,
and convulsions may also occur.
• Other clinical manifestations
include: hyperventilation, sudden
weight gain, warm, moist skin,
increased ICP: slow bounding
pulse with an increase in systolic
and decrease in diastolic
pressue and peripheral edema,
usually not marked
Hypervolemia
• Evaluating pitting edema
• Press your fingertip firmly into
the patients skin over a bony
surface for a few seconds. Then
note the depth of the imprint
your finger leaves on the skin
• A slight imprint indicates +1
pitting edema
• A deep imprint, with the skin
slow to return to its original
contour, indicates a +4
pitting edema
• When the skin resists
pressure and appears
distended, the condition is
called brawny edema, which
causes the skin to swell so
much that fluid cant be
displaced
Hypervolemia
• Diagnostic Findings:
• Decreased hematocrit resulting from
hemodilution
• Normal serum Na level
• Low serum K and BUN levels
• either due to hemodilution or higher levels
may indicate renal failure
• Low oxygen level
• Abnormal chest x-ray
• Indicates fluid accumulation
• May reveal pulmonary edema or pleural
effusions
Hypervolemia
• Treatment
• Na and fluid intake restriction
• Diuretics to promote excess fluid excretion
• Morphine and nitroglycerin (Nitro-Dur) for
pulmonary edema
• Dilate blood vessels
• Reduce pulmonary congestion and amount
of blood returning to the heart
• Digoxin for heart failure
• Strengthens cardiac contractions
Hypervolemia
• Treatment
• Supportive measures
• Oxygen administration
• Bed rest
• Hemodialysis or continuous renal
replacement therapy for renal
dysfunction
Hypervolemia
• Nursing Interventions
• Monitor fluid intake and output
• Monitor daily weight
• Monitor cardiopulmonary status
• Auscultate breathe sounds
• Assess for complaints of dyspnea
• Monitor chest x-ray results
• Monitor arterial blood gas values
• Assess for peripheral edema
• Inspect the patient for sacral edema
• Monitor infusion of I.V. solutions
• Monitor the effects of prescribed medications
General Information
• Involve destruction of the
epidermis, dermis, or subcutaneous
layers of the skin
• Can be permanently disfiguring and
incapacitating and possibly life-
threatening
General Information
• Associated imbalances result from
alterations in skin integrity and
internal body membranes, and
from effect of heat on body water
and solute loss that may result
from cellular destruction
General Information
•Type and severity of imbalance
depends on burn type and
depth, percentage body
surface area involved and burn
phase
Pathophysiology
•Burn Phase:
–Refer to stages that
describe physiologic
changes occurring
after a burn
Burn phase
Fluid-
accumulation
phase
Fluid-
remobilization
phase
Convalescent
phase
Pathophysiology
Fluid-accumulation phase:
 Last fro 36 to 48 hours after a burn
injury
 Fluid shifts from vascular
compartment to interstitial space –
third-space shift
 Edema caused by shifted fluid,
which typically reaches maximum
within 8 hours after injury
 Circulation possibly compromised
and pulses diminished from severe
edema
Burn phase
Fluid-
accumulation
phase
Fluid-
remobilization
phase
Convalescent
phase
Pathophysiology
• Several reasons for fluid
imbalances during fluid-
accumulation phase
–Damage to capillaries causing
altered vessel permeability
–Diminished kidney perfusion
–Production and release of stress
hormones such as aldosterone
and ADH
Burn phase
Fluid-
accumulation
phase
Fluid-
remobilization
phase
Convalescent
phase
Pathophysiology
Respiratory problems
Muscle and tissue injuries
GI problems
Electrolyte imbalances:
 Common during fluid accumulation
phase due to body’s hypermetabolic
needs and priority that fluid
replacement takes over nutritional
needs during emergency phase
Burn phase
Fluid-
accumulation
phase
Fluid-
remobilization
phase
Convalescent
phase
Pathophysiology
Fluid- remobilization phase :
 Also known as diuresis stage
 Starts about 48 hours after initial
burn
 Fluid shifted back to vascular
compartment
 Edema at burn site decreased, blood
flow to kidneys increased, increased
urine output
 Fluid and electrolyte imbalances can
still occur
Burn phase
Fluid-
accumulation
phase
Fluid-
remobilizatio
n phase
Convalescent
phase
Pathophysiology
Convalescent phase:
 Begins after first two phases has
been resolved
 Characterized by healing or
reconstruction of burn wound
 Major fluid shifts now resolved but
possible further fluid and electrolyte
imbalances exist as a result of
inadequate dietary intake
 Anemia is common – severe burns
typically destroy red blood cells
Burn phase
Fluid-
accumulation
phase
Fluid-
remobilization
phase
Convalescent
phase
Characteristics
• 1. Minor Burns
• Partial thickness burns are no greater than 15% of the
TBSA in the adult
• Full thickness burns are < 2% of the TBSA in the adult
• Burn areas do not involve the eyes, ears, hands, face,
feet, or perineum
• There are no electrical burns or inhalation injuries
• The client is an adult younger than 60 y.o.
• The client has no preexisting medical condition at the
time of the burn injury
• No other injury occurred with the burn
Characteristics
2. Moderate Burns
a. Partial thickness burns are deep and are 15% to 25% of
the TBSA in the adult
b. Full thickness burns are 2% to 10% of the TBSA in the
adult
c. Burn areas do not involve the eyes, ears, hands, face,
feet, or perineum
d. There are no electrical burns or inhalation injuries
e. The client is an adult younger than 60 y.o.
f. The client has no chronic cardiac, pulmonary, or
endocrine disorder at the time of the burn injury
g. No other complicated injury occurred with the burn
Characteristics
3. Major Burns
a. Partial thickness burns are > 25% of the TBSA in the
adult
b. Full thickness burns are > 10% of the TBSA
c. Burn areas involve the eyes, ears, hands, face, feet,
or perineum
d. The burn injury was an electrical or inhalation injury
e. The client is older than 60 y.o.
f. The client has a chronic cardiac, pulmonary, or
metabolic disorder at the time of the burn injury
g. Burns are accompanied by other injuries
54
Burn:Classification
Superficial (1°burns)
• Involve only the epidermal layer
of the skin.
• sunburns are commonly first-
degree burns.
55
1° burn
2° burn
56
Superficial burn (1° burn)
57
• Present of blisters indicates superficial
partial-thickness injury.
• Blister may ↑size because continuous
exudation and collection of tissue fluid.
• Healing phase of partial thickness, itching
and dryness because ↑vascularization of
sebaceous glands, ↓reduction of secretions
and ↑perspiration.
Partial thickness (2°burn)
58
2° burn
59
Partial thickness (2°burn)
60
Burn:Classification
3.Full thickness (third-degree burn)
• Destruction of the epidermis and the entire
dermis, subcutaneous layer, muscle and
bone.
• Nerve ending are destroyed-painless wound.
• Eschar may be formed due to surface
dehydration.
• Black networks of coagulate capillaries may
be seen.
• Need skin grafting because the destroyed
tissue is unable to epithelialize.
• Deep partial-thickness burn may convert to a
full-thickness burn because of infection,
trauma or ↓blood supply.
61
3° burn
62
Eschar:composed of
denatured protein
63
Full thickness (3°burn)
64
Extent of surface area burned
Rule of nines-An estimated
of the TBSA involved as a
result of a burn.
The rule of nines measures
the percentage of the body
burned by dividing the body
into multiples of nine.
The initial evaluation is made
upon arrival at the hospital.
65
Lund and Browder
• More precise method of estimating
• Recognizes that the percentage of BSA of
various anatomic parts.
• By dividing the body into very small areas and
providing an estimate of proportion of BSA
accounted for by such body parts
• Includes, a table indicating the adjustment for
different ages
• Head and trunk represent larger proportions of
body surface in children.
66
67
Age in years 0 1 5 10 15 Adult
A-head (back or
front)
9½ 8
½
6½ 5½ 4½ 3½
B-1 thigh (back or
front)
2¾ 3
¼
4 4¼ 4½ 4¾
C-1 leg (back or
front)
2½ 2
½
2¾ 3 3¼ 3½
Lund and Browder chart
TYPES OF BURNS
Thermal Burns:
caused by exposure to
flames, hot liquids, steam
or hot objects
Chemical Burns:
Caused by tissue contact
with strong alkali, or
organic compounds
Systemic toxicity from
cutaneous absorption
can occur
Radiation Burns:
caused by exposure to
UV light, x-rays, or
radioactive source
TYPES OF
BURNS
• Electrical Burns:
• Caused by heat
generated by electrical energy
as it passes through the body
• Results in internal tissue
damage
• Cutaneous burns cause
muscle and soft tissue damage
that may be extensive,
particularly in high voltage
electrical injuries
• Alternating current is
more dangerous than direct
current because it is associated
with CP arrest, ventricular
fibrillation, tetanic muscle
contractions, and long bone or
vertebral fractures
Potential
Imbalance
• Hypovolemia
• Approximately 10% of plasma
volume lost into tissue soon
after a severe burn
• Occurs because of the third
space shift causes multiple
effects:
• With burn’s damage to the
skin surface, decrease in
skins ability to prevent water
loss; patient can lose up to 8L
of fluid per day (400ml/hour)
• Potential for blood loss,
adding to fluid volume losses
Potential
Imbalance
• Hypervolemia
• Usually develops 3 to 5 days
after a major burn injury
• Occurs during the fluid
remobilization phase, as
fluid shifts from the
interstitial space back to the
vascular compartment
• May be exacerbated by
excessive administration of
I.V. fluids
Potential
Imbalance
•
•
•
•
•
Burns
NURSING PRIORITY:
The client with burn injury
is often awake, mentally alert,
and cooperative at first. The
level of consciousness may
change as respiratory status
change or as the fluid shift
occurs, precipitating
hypovolemia. If the client is
unconscious or confused,
assess him or her for the
possibility of a head injury.
Burns
• Assess for
– Patent airway
– Presence of adequate breath
sounds
– Symptoms of hypoxia
– Pulmonary damage
• Burns around the face, neck,
mouth or in the oral mucosal area
– Circulatory status
• Tachycardia and hypotension
occur early
• Elevate UO
Burns
• Assess for
– GI function – check last
time client ate
– Fluid status
• UO (30 ml/hr)
• Hypotension (< 90/60)
• Confusion /
disorientation
– Circulatory status of
the extremities
Burns
Treatment
 Respiratory status takes priority
over the treatment of the burn
injury
 If burn area is small  cold
compress or immerse in cool
water (not ice) to ↓ heat
 May have ointment on the burn
area
 Analgesics IV, IM, SQ. oral forms
may not be absorbed effectively
Burns
• Nursing intervention
– Maintain patent airway; prevent
hypoxia
– Evaluate fluid status; determine
circulatory status
– Prevent of decrease infection
– Maintain nutrition
– Prevent contractures and
scarring
– Promote acceptance and
adaptation to alterations in body
image
Burns
Formula name Electrolyte-
Containing solution
Colloid-Containing
Solution
Dextrose in
Water
Evans NSS 1 ml/kg/%burn NSS 1 ml/kg/%burn 2000 ml
Brooke LR 1.5 ml/kg/%burn 0.5 ml/kg/%burn 2000 ml
Modified Brooke LR 2 ml/kg/%burn None None
Parkland LR 4 ml/kg/%burn None None
Hypertonic
Saline
Fluid containing 250
mEq of Na/L to
maintain hourly urine
output of 70 ml in
adults
None None
First 24 hours
Burns
Formula name Electrolyte-
Containing solution
Colloid-Containing
Solution
Dextrose in
Water
Evans ½ of first 24-hr
requirement
½ of first 24-hr
requirement
2000 ml
Brooke ½ - ¾ of first 24-hr
requirement
½ - ¾ of first 24-hr
requirement
2000 ml
Modified Brooke None 0.3-0.5 ml/kg/%burn Titrate to
maintain
urine output
Parkland None 0.3-0.5 ml/kg/%burn Titrate to
maintain
urine output
Hypertonic
Saline
Same solution to
maintain hourly urine
output of 30 ml in
adults
None None
Second 24 hours
Considerations
AGE AND GENERAL HEALTH
Mortality rates are higher for children < 4 y.o, particularly
those < 1 y.o., and for clients over the age of 60 years.
Debilitating disorders, such as cardiac, respiratory,
endocrine, and renal d/o, negatively influence the client’s
response to injury and treatment.
Mortality rate is higher when the client has a pre-existing
disorder at the time of the burn injury
Electrolytes
Which one is not
a cation?
A. Calcium
B. Magnesium
C. Phosphorous
D. Sodium
Anions
and
Cations
• Anions • Cations
Bicarbonate
Chloride
Phosphorous
Calcium
Magnesium
Potassium
Sodium
WHAT DO
ELECTRO
LYTES
DO?
Controls and regulates volume of body fluids
Its concentration is the major determinant of
ECF volume
Is the chief electrolyte of ECF
Influence ICF Volume
Participates in the generation and transmission
of nerve impulses
Is an essential electrolyte in the sodium-
potassium pump
RDA: not known precisely. 500 mg
Eliminated primarily by the kidneys, smaller in
feces and perspiration
Salt intake affects sodium concentrations
Sodium is conserved through reabsorption in
the kidneys, a process stimulated by aldosterone
Normal value: 135-145 mEq/L
86
HYPONAT
REMIA
Refers to the serum sodium
concentration less than 135 mEq/L
Common with thiazide diuretic use,
but may also be seen with loop and
potassium-sparing diuretics as well
Occurs with marked sodium restriction,
vomiting and diarrhea, SIADH, etc. The
etiology may be mulfactorial
May also occur postop due to temporary alteration in
hypothalamic function, loss of GI fluids by vomiting or
suction, or hydration with nonelectrolyte solutions
Postoperative hyponatremia is a more serious
complication in premenopausal women. The reasons
behind this is unknown
Therefore monitoring serum levels is critical and careful
assessment for symptoms of hyponatremia is important
for all postoperative patients
PATHOPHYSIOLOGY
OF
HYPONATREMIA
Sodium loss from the intravascular compartment
Diffusion of water into the interstitial spaces
Sodium in the interstitial space is diluted
Decreased osmolarity of ECF
Water moves into the cell as a result of sodium loss
Extracellular compartment is depleted of water
CLINICAL SYMPTOMS
CLINICAL MANIFESTATIONS OF HYPONATREMIA
Muscle
Weakness
APATHY
Postural
hypotension
Nausea and
Abdominal
Cramps
Weight Loss
In severe hyponatremia: mental confusion, delirium, shock and coma
COLLABORATIVE CARE
MANAGEMENT
• General goal: correct sodium imbalance and restore normal fluid and
electrolyte homeostasis
• Recognition of people at risk for hyponatremia is essential for its
prevention: athletes, persons working in hot environments
• Salt is always replaced along with water
• Management includes educating vulnerable people to recognize signs
and symptoms of sodium depletion and maintaining sufficient sodium
and water intake to replace skin and insensible fluid loss
• Generally, an increased sodium and water intake provides adequate
treatment
• Education as the importance of sodium and fluid balance and the
rationale for prescription medications to ensure compliance
• Daily weight. MIO
• Monitoring of sodium levels to determine extent of replacement
• Generally, PNSS or PLRS is prescribed
• Too rapid restoration of sodium balance, hypertonic sodium solutions
may provoke brain injury
HYPERNATREMIA
• A serum sodium level above 145 mEq/L is termed hypernatremia
• May occur as a result of fluid deficit or sodium excess
• Frequently occurs with fluid imbalance
• Develops when an excess of sodium occurs without a
proportional increase in body fluid or when water loss occurs
without proportional loss of sodium
• Risk Factors: excess dietary or parenteral sodium intake, watery
diarrhea, diabetes insipidus, damage to thirst center, those with
physical or mental status compromise, and people with
hypothalamic dysfunction
PATHOPHYSIOLOGY OF
HYPERNATREMIA
Increased Sodium concentration in ECF
Osmolarity rises
Water leaves the cell by osmosis and enters
the the extracellular compartments
Dilution of fluids in ECF Cells are water depleted
Suppression of
aldosterone secretion
Sodium is exreted in the
urine
CLINICAL SYMPTOMS
CLINICAL MANIFESTATIONS
Dry, sticky mucous
membranes
Firm, rubbery
tissue turgor
Manic excitement
Tachycardia
DEATH
COLLABORATIVE CARE
MANAGEMENT
Recognition of risk
factors: bedridden and
debilitated patients,
diabetes insipidus, fluid
deprivation, the elderly
and the very young
A careful and accurate
record of MIO permits
quick recognition of
negative fluid balance
People with kidney
failure, CHF, or increased
aldosterone production
may require dietary
sodium intake restriction
Usually, osmolar balance
can be restored with oral
fluids. If not, the
parenteral route may be
necessary
Fluid resuscitation must
be undertaken with
particular caution in
patients with
compromised cardiac or
renal function
The nurse should closely
monitor the patient’s
response to fluids and
be alert to symptoms of
fluid overload
• Major cation of the ICF. Chief regulator of cellular enzyme activity and
cellular water content
• The more K, the less Na. The less K, the more Na
• Plays a vital role in such processes such as transmission of electrical
impulses, particularly in nerve, heart, skeletal, intestinal and lung tissue;
CHON and CHO metabolism; and cellular building; and maintenance of
cellular metabolism and excitation
• Assists in regulation of acid-base balance by cellular exchange with H
• RDA: not known precisely. 50-100 mEq
• Sources: bananas, peaches, kiwi, figs, dates, apricots, oranges,
prunes, melons, raisins, broccoli, and potatoes, meat, dairy products
• Excreted primarily by the kidneys. No effective conserving mechanism
• Conserved by sodium pump and kidneys when levels are low
• Aldosterone triggers K excretion in urine
• Normal value: 3.5 – 5 mEq/L
95
CAUSES AND EFFECTS OF HYPOKALEMIA
• Known as a low level of serum potassium, less than 3.5 mEq/L
Decreased Intake
↓ Food and Fluids as in
starvation
Failure to replace GI
losses
Increased Loss
↑ Aldosterone
Gastrointestinal losses
Potassium-losing diuretics
Loss from cells as in trauma,
burns
Shift of Potassium
into Cells
(No change in total
body potassium)
HYPOKALEMIA
GI Tract
Anorexia
N&V
Abdominal
distention
CNS
Lethargy,
Diminished
deep-tendon
reflexes,
Confusion,
Mental
depression
Muscles
Weakness,
Flaccid paralysis,
Weakness of
respiratory
muscles,
Respiratory arrest
CV System
Decrease in
standing BP,
Dysrhythmias,
ECG changes,
Myocardial
damage, Cardiac
arrest
Kidneys
↓Capacity to
concentrate
waste, water
loss, thirst,
kidney
damage
PATHOPHYSIOLOGY
OF
HYPOKALEMIA
= Action Potential
Nerve and Muscle Activity
Low
Extracellular
K+
Increase in
resting
membrane
potential
The cell
becomes
less
excitable
Aldosterone is secreted
Sodium is retained in the body through resorption by
the kidney tubules
Potassium is excreted
Use of certain diuretics such as thiazides and furosemide, and
corticosteroids
Increased urinary output
Loss of potassium in urine
COLLABORATIVE CARE
MANAGEMENT
• Being alert to the conditions that cause potassium depletion such as vomiting,
diarrhea and diuretics, by monitoring the patient for early warning signs
• No more than 3 enemas without consulting a physician
• Education about the importance of adequate dietary intake of potassium
• In severe hypokalemia, a patient may die unless potassium is administered
promptly
• The safest way to administer K is orally. When K is given IV, the rate of flow
must be monitored closely and should be diluted. Should not exceed 20 mEq/hr
• If PO, taken with at least ½ glass of water
• Cardiac monitoring is useful
• Potassium sparing diuretics such as triamterene, spironolactone, etc
• Symptoms of K depletion: muscle weakness, anorexia, nausea and vomiting =
appropriate referral
CAUSES AND EFFECTS OF HYPERKALEMIA
• Serum potassium level greater than 5.5 mEq/L
Excess Intake
Dietary intake of excess
of kidney’s ability to
excrete; Excess
parenteral administration
Decreased Loss
Potassium-sparing diuretics;
Renal failure; Adrenal
insufficiency
Shift of Potassium
out of the Cells
Extensive injuries,
crushing injuries,
metabolic acidosis
HYPERKALEMIA
GI Tract
N&V
Diarrhea,
Colic
CNS
Numbness,
paresthesias
Muscles
Early: irritability
Late: weakness
leading to flaccid
paralysis
CV System
Conduction
disturbance,
ventricular
fibrillation,
Cardiac Arrest
Kidneys
Oliguria
leading to
anuria
COLLABORATIVE
CARE
MANAGEMENT
• Patients at risk should be identified: impaired renal
function to avoid OTC, esp. NSAIDS which provoke
hyperkalemia; and salt substitutes that are high in
potassium
• Severity guides therapy
– Mild: Withholding provoking agent (i.e., K supp)
– Severe (>6 mEq/L: cation-exchange resin such as
Kayexalate (act by exchanging the cations in the
resin for the potassium in the intestine 
potassium is then excreted in the stool;
Continuous cardiac monitoring
• Bowel function must be maintained if Kayexelate
therapy is to be effective
• Potassium-wasting diuretics may be prescribed to
promote further potassium loss. Dialysis for
patients with renal failure to eliminate excess
potassium
• Intravenous Ca Gluconate may be prescribed to
counteract the cardiac effects of hyperkalemia
• Insulin infusions and IV NaCO3 may be used to
promote intracellular uptake of K
• Most abundant electrolyte in the body. 99% in bones and teeth
• Close link between calcium and phosphorus. High PO4, Low Ca
• Necessary for nerve impulse transmission and blood clotting and is
also a catalyst for muscle contraction and other cellular activities
• Needed for Vitamin B12 absorption and use
• Necessary for strong bones and teeth and thickness and strength of
cell membranes
• RDA: 1g for adults. Higher for children and pregnant and lactating
women according to body weight, older people, esp. post-menopausal
• Found in milk, cheese, and dried beans; some in meat and vegetables
• Use is stimulated by Vitamin D. Excreted in urine, feces, bile, digestive
secretions, and perspiration
• Normal value 8.5 – 10.5 mg/dl
102
CAUSES AND EFFECTS OF HYPOCALCEMIA
Decreased
Ionized Ca
Large
tranfusion with
citrated blood
Excess Loss
Kidney Disease
Decrease in GI Tract
and Bone Absorption
↑Magnesium
↑Calcitonin
↓Vitamin D
↓Parathyroid Hormone
HYPOCALCEMIA
Bones
Osteoporosis
leading to
Fractures
CNS
Tingling
↓
convulsions
Other
Abnormal
deposits
of calcium
in body
tissues
Muscles
Muscle spasm
↓
Tetany
Cardiovascular
System
Dysrhythmias
↓
Cardiac arrest
Inadequate
Intake
Dietary Deficit
PATHOPHYSIOLOGY OF HYPOCALCEMIA
•Calcium ions are thought to line the
pores of cell membranes, especially
neurons
•Calcium and Sodium repel each other
•When serum calcium levels are low, this
blocking effect is minimized
•When Sodium moves more easily into
the cell, depolarization takes place more
easily
•This results in increased excitability of
the nervous system leading to muscle
spasm, tingling sensations, and if severe,
convulsions and tetany
•Skeletal, smooth, and cardiac muscle
functions are all affected by
overstimulation
Sodium Calcium
CLINICAL MANIFESTATIONS OF HYPOCALCEMIA
COMPLAINT OF NUMBNESS AND TINGLING OF EARS, NOSE,
FINGERTIPS OR TOES
TREATMENT
PAINFUL MUSCULAR SPASMS (TETANY)
ESPECIALLY OF FEET AND HANDS (CARPOPEDAL
SPASMS), MUSCLE TWITCHING AND CONVULSIONS
MAY FOLLOW
TESTS USED TO ELICIT SIGNS OF CALCIUM DEFICIENCY
COLLABORATIVE CARE
MANAGEMENT
• Identify risk factors: Inadequate calcium intake, excess calcium loss, Vitamin D deficiency,
patients with poor diets
• Education about the importance of adequate calcium and Vitamin D intake
• Patients undergoing thyroid, parathyroid, and radical neck surgery are particularly
vulnerable to hypocalcemia secondary to parathyroid hormone deficit
• Monitoring of serum calcium levels and correction of deficits
• Citrate is added to store blood to prevent coagulation.
• Citrate + Transfusion = Citrate+Calcium
• Normally, Liver + Citrate = Quick metabolism
• Preexisting calcium deficit/hepatic dysfunction/large amounts of BT very rapidly =
hypocalcemia
• With acute hypocalcemia, Ca Gluconate is used + Continuous cardiac monitoring
• Mild Hypocalcemia: High calcium diet or oral calcium salts
• If PTH or Vit D Deficiency is the cause: aluminum hydroxide gel is used because when serum
phosphate level rises, calcium level falls
• Complication: Bone demineralization
• Therefore, careful ambulation should be encouraged to minimize bone resorption
HYPERCALCEMIA: Serum concentration > 10mg/dL
Causes and Effects
Loss from bones
Immobilization,
Carcinoma with bone
metastases, Multiple
myeloma
Excess Intake
↑ Calcium diet (esp. milk)
Antacids containing calcium
Increase in factors
Causing Mobilization
from bone
↑PTH, ↑ Vitamin D,
steroid therapy
HYPERCALCEMIA
Kidneys
Stones
↓
Kidney
Damage
CNS
↓Deep-tendon
reflexes
↓
Lethargy
↓
Coma
Bones
Bone pain
↓
Osteoporosis
↓
Fractures
Muscles
Muscle fatigue,
hypotonia
↓
↓ GI motility
CV System
Depressed
activity
↓
Dysrhythmias
↓
Cardiac Arrest
HOW IT
HAPPENS
HYPERCALCEMIA
DEPRESSED NERVE
AND MUSCLE ACTIVITY
DEEP TENDON
REFLEXES MAY BE
DECREASED OR
ABSENT
MYOCARDIAL
FUNCTION IS ALTERED
CLINICAL MANIFESTATIONS OF HYPERCALCEMIA
Decreased GI
Motility
Cardiac Dysrhythmias
Constipation
Nausea
Mental status changes:
lethargy, confusion,
memory loss
CLINICAL MANIFESTATIONS OF HYPERCALCEMIA
Immobilization Bone
Demineralization
Calcium
accumulates in
the ECF and
passes through
the kidneys
Ca Precipitation
Calcium Stones
COLLABORATIVE CARE
MANAGEMENT
• Mild hypercalcemia: hydration and education about avoiding foods
high in calcium or medications that promote calcium elevation
• Ambulation as appropriate; weight-bearing exercises as tolerated
• Trapeze, resistance devices
• Marked hypercalcemia: prevention of pathologic fractures,
individualized plan of care
• Prevention of renal calculi: encourage oral fluids to prevent
concentrated urine: 3000 to 4000 mL/day unless contraindicated
• Acid-ash fruit juices: cranberry juice and prune juice
• Severe hypercalcemia: medical emergency: continuous cardiac
monitoring, hydration, IV furosemide, Calcitonin and/or plicamycin
(mithramycin), q2 serum and urinary electrolytes
• Mostly found within body cells: heart, bone, nerve, and muscle tissues
• Second most important cation in the ICF, 2nd to K+
• Functions: Metabolism of CHO and CHON, protein and DNA synthesis,
DNA and RNA transcription, and translation of RNA, maintains normal
intracellular levels of potassium, helps maintain electric activity in nervous
tissue membranes and muscle membranes
• RDA: about 18-30 mEq; children require larger amounts
• Sources: vegetables, nuts, fish, whole grains, peas, and beans
• Absorbed in the intestines and excreted by the kidneys
• Plasma concentrations of magnesium range from 1.5 – 2.5 mEq/L, with
about one third of that amount bound to plasma proteins
113
HYPOMAGNESEMIA: Serum level < 1.5 mEq/L
• Usually coexists with hypokalemia and less often with hypocalcemia
Decreased Intake
Prolonged
malnutrition,
Starvation
Impaired absorption from GI Tract
Malabsorption syndrome, Alcohol Withdrawal
Syndrome, Hypercalcemia, Diarrhea,
Draining gastrointestinal fistula
Excessive
Excretion
↑Aldosterone,
Conditions
causing large
losses of urine
HYPOMAGNESEMIA
Mental Changes
Agitation,
Depression,
Confusion
CNS
Convulsions,
Paresthesias,
Tremor, Ataxia
Muscles
Cramps,
Spasticity, Tetany
CV System
Tachycardia,
Hypotension,
Dysrhythmias
HYPOKALEMIA
PATHOPHYSIOLOGY OF
HYPOMAGNESEMIA
Low serum magnesium
level
Increased acetylcholine release
Increased neuromuscular irritability
Increased sensitivity to acetylcholine at the myoneural
junction
Diminished threshold of
excitation for the motor nerve
Enhancement of myofibril
contraction
PATHOPHYSIOLOGY OF
HYPOMAGNESEMIA
High Serum Calcium
Increased acetylcholine
release
Increased neuromuscular irritability
Increased sensitivity to acetylcholine at the myoneural
junction
Diminished threshold of
excitation for the motor nerve
Enhancement of myofibril
contraction
High Serum Calcium
Excretion of Magnesium
By the GI tract
PATHOPHYSIOLOGY OF HYPOMAGNESEMIA
MAGNESIUM
INHIBITS TRANSPORT OF PTH
DECREASE IN THE AMOUNT OF CALCIUM BEING RELEASED
FROM THE BONE
POSSIBLE CALCIUM DEFICIT
CLINICAL MANIFESTATIONS OF HYPOMAGNESEMIA
CONFUSION
DEPRESSION
CRAMPS
TETANY CONVULSIONS
COLLABORATIVE CARE
MANAGEMENT
• Recognition of people at risk: people taking loop diuretics and digoxin
should be encouraged to eat foods rich in magnesium, such as fruits,
vegetables, cereals, and milk
• Recognition of signs and symptoms of magnesium deficiency
• Magnesium is essential for potassium resorption, so if hypokalemia
does not respond to potassium replacement, hypomagnesemia should
be suspected
• Treatment of the underlying cause is the first consideration in
hypomagnesemia
• Severe: parenteral magnesium replacement is indicated
• IV therapy: continuous cardiac monitoring
• Safety measures for patients with mental status changes
HYPER
MAGNE
SEMIA:
Serum
Mg
level
2.5
mEq/L
• Seldom develops in the
presence of normal renal
function
• May occur as a result of Mg
replacement
• May occur when MgSO4 is
administered to prevent
seizures resulting from
eclampsia
• Careful monitoring is
imperative
PATHOPHYSIOLOGY
Renal failure, Excessive IV infusion of
magnesium, Decreased GI elimination and/or
absorption, etc.
Accummulation of Mg in the body
Diminishing of reflexes, drowsiness, lethargy
Mg Level Rises
Severe Respiratory Depression
RESPIRATORY ARREST may occur
Altered Electrical Conduction
Slowed heart
rate and AV
Block
Peripheral
vasodilation
Hypotension, flushing, and
increased skin warmth
COLLABORATIVE CARE
MANAGEMENT
• Identification of patients at risk: those with impaired renal
function to avoid OTC that contain magnesium such as Milk of
Magnesia and some Mg-containing antacids
• Any patient receiving parenteral magnesium therapy should be
assessed frequently for signs of hypermagnesemia
• Mild hypermagnesemia: withholding magnesium-containing
medications may suffice
• Renal failure: dialysis
• Severe: may require treatment with calcium gluconate (10-20
mL of 10% Ca Gluconate administered over 10 minutes)
• If cardiorespiratory collapse is imminent, the patient may
require temporary pacemaker and ventilator support
NURSING
MANAGEMENT
OF PATIENT
WITH FLUID
AND
ELECTROLYTE
IMBALANCES
Parameter_____Fluid Excess___ Fluid Loss/Electrolyte Imbalance____
Behavior Tires easily; Change in behavior, confusion, apathy
Head, neck Facial edema, distended neck Headache, thirst, dry mucous
membranes veins
Upper GI Anorexia, nausea, vomiting
Skin Warm, moist, taut, cool feeling Dry, decreased turgor where
edematous
Respiration Dyspnea, orthopnea, productive Changes in rate and depth of respiration
cough, moist breath sounds
Circulation Loss of sensation in edematous Pulse rate changes, dysrhythmia,
postural areas, pallor, bounding pulse,
increased blood pressure hypotension
Abdomen Increased girth, fluid wave Distention, abdominal cramps
Elimination Constipation Diarrhea, constipation
Extremities Dependent edema, “pitting” Muscle weakness, tingling, tetany ,
discomfort from weight of
bedclothes
Pitting edema
Dependent edema
Refractory Edema
LABORATORY VALUES
FLUID DEFICIT FLUID EXCESS
Hemoconcentration Hemodilution
↑ Hct, BUN, E+ levels ↓ Hct, BUN, E+ levels
↑ Urine Specific Gravity ↓ Urine Specific Gravity
Determined from analysis of patient data
Diagnostic Title Possible Etiologic Factors
1 Deficient fluid volume Active fluid volume loss
(hemorrhage, diarrhea, gastric
intubation, wounds, diaphoresis),
inadequate fluid intake, failure of
regulatory mechanisms,
sequestration of body fluids
2 Excess Fluid Volume Excess fluid intake, excess sodium
intake, compromised regulatory
processes
EXPECTED PATIENT OUTCOMES
1. Will maintain functional fluid volume as evidenced by
adequate urinary output, stable weight, normal vital
signs, normal urine specific gravity, moist mucus
membranes, balanced intake and output, elastic skin turgor,
prompt capillary refill, and absence of edema
2. Will verbalize understanding of treatment plan and
causative factors that led to the imbalance
1,2Intake and Output Monitoring
- Type and amount of fluid the patient has received and the
route by which they were administered
- Record of solid food intake. Gelatin or Popsicles are
recorded as fluids
- Ice chips are recorded by dividing the amount of chips
by ½ (60 mL of chips = 30 mL water)
- Accurate output record and described by color, content,
and odor (Normally, gastric contents are watery and pale
yellow-green; they usually have a sour odor)
- With acid-base balance upset, gastric secretions may
have a fruity odor because of ketone bodies
- Bile: thicker than gastric juice, dark green to brown,
acrid odor, bitter taste when vomiting
- NGT irrigation added to intake
- Stools: difficult to estimate amount; consistency, color,
and number of stools provide a reasonable estimate
- Peritoneal or pleural fluid drainage is recorded as output
as with its amount, color, and clarity
- Character and volume of urine. Place signs and
materials so that an accurate record of UO is maintained
1,2 Intake and Output Monitoring
- Evaluate and refer urine specific gravity as appropriate
(normal value is 1.003 – 1.030). The implications are:
High Dehydration
Low SIADH, overhydration
- Drainage, fluid aspirated from any body cavity must be
measured. With dressings, fluid loss is the difference
between the wet dressings and the dry weight of the
dressing
- Accurate recording of the temperature to help the
physician determine how much fluid should be replaced
1,2 Daily Weight
- Evaluate trends in weight (An increase in 1kg in weight
is equal to the retention of 1L of fluid in an edematous
patient)
Considerations:
- Daily weights early in the morning after voiding
but before he or she has eaten or defecated
1 Replacement of Fluid and Electrolytes
General Principles:
- Either by oral intake (healthiest way), tube feeding,
intravenous infusion, and/or total parenteral nutrition
- Normal saline solution and plain water should also be
given by slow drip to replace daily fluid loss
- IV administration per doctor’s orders
- Fluid replacement considerations:
* Most effective when apportioned over 24 hr period
(Better regulation, ↓potential for calculi formation and
subsequent renal damage, ↓potential for circulatory
overload which may cause in fluid and electrolyte
shifts)
* Administer concentrated solutions of Na, Glucose or
protein because they require body fluids for dilution
* Consider the size of the patient (small adult has less
fluid in each compartment, especially in the
intravascular compartment)
- Promote oral intake as appropriate
* Caution with coffee, tea, and some colas
* small amount at frequent intervals is more useful than a
large amount presented less often
* Always give consideration to cultural and aesthetic
aspects of eating
- Give mouth care to a dehydrated patient before and after meals
and before bedtime (Xerostomia may lead to disruption of t
issues in the oral cavity)
- Avoid irritating foods
- Stimulation of saliva may be aided by hard candy or chewing
gum or carboxymethylcellulose (artificial saliva)
- Keep lips moist and well lubricated
- Give salty broth or soda crackers for sodium replacement and
tea or orange juice for potassium replacement as appropriate.
Bananas, citrus fruits and juices, some fresh vegetables,
coffee, and tea are relatively high in potassium and low in
sodium. Milk, meat, eggs, and nuts are high in protein,
sodium and potassium.
- Offer milk for patients with draining fistulas from any portion of the
GI tract. Lactose intolerance is not necessarily a
contraindication (Lactase enzyme preparations are available)
- Increase usual daily requirement of foods when losses must be
restored, as tolerated
* Patients with cardiac and renal impairments are
instructed to avoid foods containing high levels
of sodium, potassium and bicarbonate
- Administer replacement solutions through tube feeding as is
* Either water, physiologic solution of NaCl, high protein
liquids, or a regular diet can be blended, diluted and
given by gavage
* The water content in the tube feeding needs to be
increased if:
1 the patient complains of thirst
2 the protein or electrolyte content of the tube
feeding is high
3 the patient has fever or disease causing an
increased metabolic rate
4 UO is concentrated
5 signs of water deficit develop
- Administer parenteral fluids as necessary
* Types of solutions
- D5W (hypotonic) is given short-term for hyponatremia
- D5NSS may be given depending on the serum levels of
sodium and vascular volume + KCl to meet normal
intake needs and replace losses for hyponatremia
- Dextrose 5% in 0.2% normal saline is generally used as
a maintenance fluid
- Dextrose 5% in ½ normal saline is generally used as a
replacement solution for losses caused by
gastrointestinal drainage
- PNSS is given primarily when large amounts of sodium
have been lost and for patients with hyponatremia
- LRS is also isotonic because it remains in the
extracellular space
- Fructose or 10-20% glucose in distilled water are
hypertonic solutions and may partially meet body
needs for CHOs
- Dextran (commonly-used plasma expander) increases
plasma volume by increasing oncotic pressure. May
cause prolonged bleeding time and is CI in patients
with renal failure, bleeding disorders, or severe CHF
* Administration
- The rate should be regulated according to the patient’s
needs and condition per doctor’s orders
- Monitor UO carefully. Refer marked decreases!
- Verify orders for potassium administration in patients
with renal failure and untreated adrenal insufficiency
- Usual rate for fluid loss replacement: 3ml/min
- Recognize signs of pulmonary edema (bounding pulse,
engorged peripheral veins, hoarseness, dyspnea,
cough, and rales) that can result from ↑IV rate
- If infiltration occurs, the infusion should be stopped
immediately and relocated. Peripheral IV sites are
generally rotated every 72 hours
- For dextran and other plasma expanders, observe for
anaphylactic reaction (apprehension, dyspnea,
wheezing, tightness of chest, angioedema,
itching, hives and hypotension). If this happens,
switch infusion to nonprotein solution and run at KVO
rate, notify physician and monitor VS
- Pronounced and continued thirst despite administration
of fluids is not normal and should be reported (may
indicate DM or hypercalcemia)
* Patient/Family Education
- Include the signs and symptoms of water excess in
discharge instructions
- With drug therapy, instruct patient and family regarding
correct method of administration, correct dose, and
therapeutic and adverse effects
- Instruct to read labels for nutritional content
* For K restriction: avoid organ meats, fresh and dried
fruits, and salt substitutes
- Skin assessment and care, positioning techniques for
patients with mobility restrictions
* Achievement of outcomes is successful in disturbances in fluid
and electrolyte balance:
1 Maintains functional fluid volume level with adequate UO,
VS within the patient’s normal limits, sp gr of urine
within 1.003-1.035, moist mucous membranes, stable
weight, Intake=output, elastic skin turgor, and no edema
2 States possible causes of imbalance and plan to prevent
recurrence of imbalances
3 Reports a decrease or absence of symptoms causing
discomfort
Fluids and Electrolytes
Acid-base
balance
DRAWING ARTERIAL BLOOD GASES
ALLEN’S TEST
ARTERIAL PUNCTURE
BASIC REGULATION OF ACID-BASE BALANCE
CO2 + H2O ↔ H2CO3 ↔ H+ + HCO3
The lungs help control acid-base balance by blowing off or
retaining CO2. The kidneys help regulate acid-base balance by
excreting or retaining HCO3
TYPES OF ACID-BASE DISTURBANCES
Depression of the central
nervous system, as
evidenced by disorientation
followed by coma
Overexcitability of the
nervous system; muscles
may go into a state of tetany
and convulsioons
Compensation
RESPIRATORY ACIDOSIS: CARBONIC ACID EXCESS
Damage to the respiratory center in the medulla, drug or narcotic use, obstruction
of respiratory passages, respiratory and respiratory muscle disorders
Decrease in the rate of pulmonary ventilation
Increase in the concentration of CO2, carbonic acid,
and hydrogen ions
RESPIRATORY ACIDOSIS
Potassium moves out of the cells
HYPERKALEMIA
VENTRICULAR FIBRILLATION
NURSING MANAGEMENT OF
RESPIRATORY ACIDOSIS
ASSESSMENT
* Health Hx: complaints of headache, confusion, lethargy,
nausea, irritability, nausea, irritability, anxiety, dyspnea, and
blurred vision, preexisting conditions
* Physical Examination: lethargy to stupor to coma, tachycardia,
hypertension, cardiac dysrhythmias, airway patency
NURSING DIAGNOSES include but are not limited to:
• Diagnostic Title Possible Etiologic Factors
1 Impaired gas exchange Hypoventilation
2 Disturbed thought processes Central nervous system depression
3 Anxiety Hypoxia, hospitalization
4 Risk for ineffective family Illness of a family member
coping
5 Ineffective airway clearance Hypoventilation, secretions
6 Ineffective breathing pattern Hypoventilation, dyspnea
NURSING MANAGEMENT OF
RESPIRATORY ACIDOSIS
EXPECTED PATIENT OUTCOMES include but are not limited to:
1 Will maintain airway patency and adequate breathing
rate and rhythm will return of ABGs to patient’s
normal level
2 Will be alert and oriented to time, place, and person,
or to his or her normal baseline level of
consciousness
3 Will cope with anxiety
4 Will exhibit effective coping and awareness of
effective support systems
5 Will have secretions that are normal for self in amount
and can be raised
6 Will maintain adequate rate and depth of respirations
using pursed lip and other breathing
techniques when necessary (as in the patient
with COPD)
NURSING MANAGEMENT OF PATIENT
WITH RESPIRATORY ACIDOSIS
INTERVENTIONS
1 Supporting effective gas exchange
- Provide a position of comfort to allow ease of
respiration
- Obtain and monitor ABG results and VS. Refer
accordingly
- Provide and monitor supplemental oxygen as
ordered
- Turn the patient q2 and PRN
- Provide pulmonary hygiene PRN
- Maintain adequate hydration
- Provide comfort measures such as mouth care
- Assist with ADLs
- Instruct patient regarding coughing and deep
breathing and management of disease condition,
especially COPD
2 Coping with disturbed thought processes
- Do frequent neurologic assessments
- Monitor and document person’s baseline LOC
frequently
NURSING MANAGEMENT OF PATIENT WITH RESPIRATORY ACIDOSIS
- Reorient as necessary by providing calendars, clocks, etc.
3 Relieving anxiety
- Provide a calm, relaxed environment
- Give clear, concise explanations of treatment plans
- Encourage expression of feelings
- Provide support and information to patient and family
- Teach relaxation techniques
- Assist the patient to identify coping mechanisms to deal with
anxiety and stress
4 Enhancing coping mechanisms
- Provide support and information to family members about the
patient’s ongoing condition
- Reassure them that there is a physiologic cause for the
patient’s behavior
NURSING MANAGEMENT OF PATIENT WITH RESPIRATORY ACIDOSIS
- Encourage questions and open communication
5 Promote airway clearance
- Implement regular breathing and coughing exercises
- Do suctioning as necessary
- Maintain good hydration
- Do chest physiotherapy as appropriate
6 Promoting an effective breathing pattern
- Maintain alveolar ventilation
- Teach the patient proper breathing techniques as well as panic
control breathing
NURSING MANAGEMENT OF PATIENT WITH RESPIRATORY ACIDOSIS
EVALUATION. Achievement of outcomes is successful when the patient:
1a. Demonstrates improved ventilation and oxygenation
1b Has vital signs, ABGs, and cardiac rhythm within own normal
range
2 Returns to baseline LOC
3 Reports reduced anxiety
4 Family uses adequate coping mechanisms
5 Is able to raise secretions on own
6 Demonstrate effective breathing techniques
RESPIRATORY ALKALOSIS: CARBONIC ACID DEFICIT
Anxiety, hysteria, fever, hypoxia, pain, pulmonary disorders, lesions
affecting the respiratory center in the medulla, brain tumor,
encephalitis, meningitis, hyperthyroidism, gram-negative sepsis
Hyperventilation: Excessive pulmonary ventilation
Decrease in hydrogen ion concentration
RESPIRATORY ALKALOSIS
NURSING MANAGEMENT OF RESPIRATORY ALKALOSIS
ASSESSMENT
* Health Hx: anxiety, shortness of breath, muscle cramps or
weakness, palpitations, panic, dyspnea
* Physical Examination: light-headedness, confusion as a
result of cerebral hypoxia, hyperventilation, tachycardia
or arrhythmia, muscle weakness, (+) Chvostek’s sign or
Trousseau’s sign indicating a low ionized serum calcium
level secondary to hyperventilation and alkalosis,
hyperactive deep tendon reflexes, unsteady gait, muscle
spasms to tetany, agitation, psychosis, seizures in
extreme cases, decreased potassium levels
NURSING DIAGNOSES include but are not limited to:
Diagnostic Title Possible Etiologic Factors
1 Anxiety Stress, fear
2 Ineffective breathing pattern Hyperventilation, anxiety
3 Disturbed thought processes CNS excitability; irritability
4 Risk for injury Change in LOC, and potential for
seizures
NURSING MANAGEMENT OF RESPIRATORY ALKALOSIS
EXPECTED PATIENT OUTCOMES include but are not limited to:
1 Will report decreased anxiety; verbalizes methods to
cope with anxiety
2 Will return to normal respiratory rate and rhythm or at
least decreased hyperventilation, with return to
baseline ABGs
3 Will exhibit reorientation to person, place, and time as
per patient’s baseline
4 Will be free from injury
INTERVENTIONS
1 Allay anxiety
- Give antianxiety medications as
ordered
- Have patient breath into a paper bag
- Teach relaxation techniques when initial
anxiety attack is over
NURSING MANAGEMENT OF PATIENT WITH RESPIRATORY ACIDOSIS
INTERVENTIONS
2 Promoting an Effective Breathing Pattern
- Encourage the patient to slow his or her RR
- Maintain a calm and comforting attitude
- Position the patient to promote maximal ease of
inspiration
- Assist the patient with relaxation techniques
3 Coping with Disturbed Thought Processes
- Do frequent reorientation
- Encourage family to participate in patient’s care
- Use simple, direct statements or directions
- Allow the patient adequate time to respond
4 Preventing injuries
- Perform neurologic assessment frequently and
document
- Institute safety and seizure precautions
- Assess frequently for muscle strength and
coordination
NURSING MANAGEMENT OF PATIENT WITH RESPIRATORY ACIDOSIS
EVALUATION. Achievement of outcomes is successful when the patient:
1 Reports reduction in anxiety levels
2a Demonstrates effective normal breathing patterns
2b Has ABG results within patient’s normal baseline
3 Returns to normal baseline LOC and orientation level
4 Remains free from injury; no seizure activity
METABOLIC ACIDOSIS: BICARBONATE DEFICIT
Increased acid production, uncontrolled diabetes mellitus,
alcoholism, starvation, renal acidosis, lactic acidosis, increased acid
ingestion, ethanol, salicylates, loss of bicarbonate, severe diarrhea,
intestinal fistulas, adrenal insufficiency, hypoparathyroidism
Excess organic acids are added to body fluids or
bicarbonate is lost
Decrease in bicarbonate concentration
METABOLIC ACIDOSIS
NURSING MANAGEMENT OF METABOLIC ACIDOSIS
ASSESSMENT
* Health Hx: anorexia, nausea, vomiting, abdominal pain,
headache, thirst if the patient is dehydrated
* Physical Examination: confusion, hyperventilation, warm,
flushed skin, bradycardia and other dysrhythmias,
decreasing LOC, nausea, vomiting, diarrhea, Kussmaul
respirations, and acetone breath, especially if acidosis is
due to ketoacidosis. Symptoms may progress to coma if
untreated
NURSING DIAGNOSES include but are not limited to:
Diagnostic Title Possible Etiologic Factors
1 Disturbed thought processes Secondary to CNS depression
2 Decreased cardiac output Dysrhythmias
3 Risk for injury Secondary to altered mental state
4 Risk for imbalanced fluid Diarrhea, renal failure
volume
NURSING MANAGEMENT OF METABOLIC ACIDOSIS
EXPECTED PATIENT OUTCOMES include but are not limited to:
1 Will return to usual baseline LOC
2 Will return to normal baseline parameters for vital
signs with improved CO and decreased or resolved
dysrhythmias
3 Will remain in a safe, secure environment without
injury
4 Will maintain fluid and electrolyte balance and stable
renal status
INTERVENTIONS
1 Coping with disturbed thought processes
- Monitor LOC and reorient as necessary
- Monitor VS, esp. RRR, BP, and T
- Monitor ABGs to assess the effects of
treatment
NURSING MANAGEMENT OF PATIENT WITH METABOLIC ACIDOSIS
2 Supporting cardiac output
- Monitor VS, MIO, and fluid and electrolyte
balance
- Institute cardiac monitoring to evaluate
cardiac status
3 Promoting safety
- Provide a safe, secure and monitored
environment
- Institute safety precautions
4 Promoting return of fluid and electrolyte balance
- Monitor MIO
- Administer medications per medical order
NURSING MANAGEMENT OF PATIENT WITH METABOLIC ACIDOSIS
EVALUATION. Achievement of outcomes is successful when the
patient:
1 Exhibits baseline-level consciousness and orientation
2 Returns to normal baseline parameters for vital signs and
Cardiac Output with cardiac dysrhythmias resolved
3 Remains free from injury
4 Maintains fluid and electrolyte balance and stable renal function
METABOLIC ALKALOSIS: BICARBONATE EXCESS
Loss of stomach acid, gastric suctioning, persistent vomiting, excess
alkali intake, intestinal fistulas, hypokalemia, Cushing’s syndrome or
aldosteronism, potassium-diuretic therapy
Excessive amounts of acid substance and
hydrogen ions are lost from the body or large
amounts of bicarbonate or lactate are added orally
or IV
Excess of base elements
METABOLIC ALKALOSIS
NURSING MANAGEMENT OF METABOLIC ALKALOSIS
ASSESSMENT
* Health Hx: Prolonged vomiting or nasogastric suctioning,
frequent self-induced vomiting, muscle weakness, light-
headedness, ingestion of large amounts of licorice or
antacids, use of diuretics, muscle cramping, twitching, or
tingling
* Physical Examination: mental confusion, dizziness, changes in
LOC, hyperreflexia, tetany, dysrhthmias, seizurees, respiratory
failure, positive Chvostek’s or Trosseau’s sign if the patient has
a low ionized serum calcium level, decreased hand grasps,
generalized muscle weakness, decreased serum calcium or
potassium level, impaired concentration, seizures, ECG changes
consistent with hypokalemia
NURSING DIAGNOSES include but are not limited to:
Diagnostic Title Possible Etiologic Factors
1 Disturbed thought processes CNS excitation
2 Decreased cardiac output Dysrhythmias and electrolyte
imbalances
3 Risk for injury Muscle weakness, tetany,
confusion and possible seizures
4 Risk for imbalanced fluid volume Nasogastric drainage, diuretic
therapy volume
NURSING MANAGEMENT OF METABOLIC ALKALOSIS
EXPECTED PATIENT OUTCOMES include but are not limited to:
1 Will return to usual baseline LOC and orientation
2 Will return to normal baseline parameters for vital
signs with improved CO with resolution of electrolyte
imbalances and decreased or resolved cardiac
dysrhythmias
3 Will remain in a safe, secure environment without
injury
4 Will maintain fluid and electrolyte balance
INTERVENTIONS
1 Coping with disturbed thought processes
- Monitor LOC and reorient as necessary
- Monitor VS, esp. RRR, BP, and T
- Monitor ABGs to assess the effects of
treatment
- Institute cardiac monitoring as ordered
NURSING MANAGEMENT OF PATIENT WITH METABOLIC ALKALOSIS
2 Supporting cardiac output
- Monitor VS, MIO, and fluid and electrolyte
balance
- Institute cardiac monitoring to evaluate
cardiac status
3 Promoting safety
- Provide a safe, secure and monitored
environment
- Institute safety precautions
4 Promoting return of fluid and electrolyte balance
- Monitor MIO
- Administer medications per medical order
NURSING MANAGEMENT OF PATIENT WITH METABOLIC
ALKALOSIS
EVALUATION. Achievement of outcomes is successful when the
patient:
1 Manifests mental status has returned to baseline
2 Is free from cardiac dysrhythmias
3 Remains free from injury
4 Maintains fluid balance at baseline level
CRITICAL THINKING EXERCISES
A 32-year-old administrative assistant comes to the urgent care
center with a 72-hour history of vomiting secondary to influenza.
She is lethargic and states, “My muscles are twitching.” Her RR is
18/min and HR is 110 bpm, T=100.4F. Her blood pressure is 110/68
which she states “is about normal for me.” Her ABG values are as
follows:
pH: 7.57
PaO2: 92
PaCO2: 41
HCO3: 36
Describe her acid-base status, probable cause for the imbalance
and treatment

More Related Content

What's hot

Fluid and electrolyte balance
Fluid and electrolyte balanceFluid and electrolyte balance
Fluid and electrolyte balance
Diwakar vasudev
 
Fluid & Electrolytes Balance
Fluid & Electrolytes  BalanceFluid & Electrolytes  Balance
Fluid & Electrolytes Balancemohammed indanan
 
Fluids and electrolytes ppt
Fluids and electrolytes pptFluids and electrolytes ppt
Fluids and electrolytes ppt
rajat1906
 
Fluid & Electrolyte balance by Dr Nesar
Fluid & Electrolyte balance by Dr NesarFluid & Electrolyte balance by Dr Nesar
Fluid & Electrolyte balance by Dr Nesar
Student
 
Intake and output class of year one student final 2
Intake and output class of year one student final 2Intake and output class of year one student final 2
Intake and output class of year one student final 2
aku karachi
 
Fluid and electrolytes
Fluid and electrolytes Fluid and electrolytes
Fluid and electrolytes
Mohammed Haneef Farooq
 
Type and indication of iv therapy 2 (1) 2
Type and indication of iv therapy 2 (1) 2Type and indication of iv therapy 2 (1) 2
Type and indication of iv therapy 2 (1) 2
HI HI
 
fluid and electrolyte
 fluid and electrolyte fluid and electrolyte
fluid and electrolyte
Bryan Castanares
 
Fluid and electrolytes, acid base balance
Fluid and electrolytes, acid base balanceFluid and electrolytes, acid base balance
Fluid and electrolytes, acid base balance
marudhar aman
 
Fluid and electrolyte balance
Fluid and electrolyte balanceFluid and electrolyte balance
Fluid and electrolyte balancePhey Yaaz
 
Intake and output chart
Intake and output chartIntake and output chart
Intake and output chart
SusmitaBarman2
 
Iv fluids
Iv fluidsIv fluids
Iv fluids
Ekta Patel
 
Nsg care with Fluid & Electrolyte imbalance.pptx
Nsg care with Fluid & Electrolyte imbalance.pptxNsg care with Fluid & Electrolyte imbalance.pptx
Nsg care with Fluid & Electrolyte imbalance.pptx
Abhishek Joshi
 
Fluid and electrolyte balance
Fluid and electrolyte balanceFluid and electrolyte balance
Fluid and electrolyte balance
Dr B Naga Raju
 
Fluids, Electrolytes & IV Therapy
Fluids, Electrolytes & IV TherapyFluids, Electrolytes & IV Therapy
Fluids, Electrolytes & IV Therapychrissie argana
 
FLUID THERAPY IN NURSING
FLUID THERAPY IN NURSINGFLUID THERAPY IN NURSING
FLUID THERAPY IN NURSING
jones H.M Munang'andu(MBA)
 
Pressure Ulcer Prevention & Skin Care
Pressure Ulcer Prevention & Skin CarePressure Ulcer Prevention & Skin Care
Pressure Ulcer Prevention & Skin Care
NAW52
 

What's hot (20)

Fluid and electrolyte balance
Fluid and electrolyte balanceFluid and electrolyte balance
Fluid and electrolyte balance
 
Fluid & Electrolytes Balance
Fluid & Electrolytes  BalanceFluid & Electrolytes  Balance
Fluid & Electrolytes Balance
 
Ms.fluid&electrolytes
Ms.fluid&electrolytesMs.fluid&electrolytes
Ms.fluid&electrolytes
 
Fluids and electrolytes ppt
Fluids and electrolytes pptFluids and electrolytes ppt
Fluids and electrolytes ppt
 
Fluid, electrolyte, and acid base balance.drjma
Fluid, electrolyte, and acid base balance.drjmaFluid, electrolyte, and acid base balance.drjma
Fluid, electrolyte, and acid base balance.drjma
 
Fluid & Electrolyte balance by Dr Nesar
Fluid & Electrolyte balance by Dr NesarFluid & Electrolyte balance by Dr Nesar
Fluid & Electrolyte balance by Dr Nesar
 
Fluid And Electrolytes1
Fluid And Electrolytes1Fluid And Electrolytes1
Fluid And Electrolytes1
 
Intake and output class of year one student final 2
Intake and output class of year one student final 2Intake and output class of year one student final 2
Intake and output class of year one student final 2
 
Fluid and electrolytes
Fluid and electrolytes Fluid and electrolytes
Fluid and electrolytes
 
Type and indication of iv therapy 2 (1) 2
Type and indication of iv therapy 2 (1) 2Type and indication of iv therapy 2 (1) 2
Type and indication of iv therapy 2 (1) 2
 
fluid and electrolyte
 fluid and electrolyte fluid and electrolyte
fluid and electrolyte
 
Fluid and electrolytes, acid base balance
Fluid and electrolytes, acid base balanceFluid and electrolytes, acid base balance
Fluid and electrolytes, acid base balance
 
Fluid and electrolyte balance
Fluid and electrolyte balanceFluid and electrolyte balance
Fluid and electrolyte balance
 
Intake and output chart
Intake and output chartIntake and output chart
Intake and output chart
 
Iv fluids
Iv fluidsIv fluids
Iv fluids
 
Nsg care with Fluid & Electrolyte imbalance.pptx
Nsg care with Fluid & Electrolyte imbalance.pptxNsg care with Fluid & Electrolyte imbalance.pptx
Nsg care with Fluid & Electrolyte imbalance.pptx
 
Fluid and electrolyte balance
Fluid and electrolyte balanceFluid and electrolyte balance
Fluid and electrolyte balance
 
Fluids, Electrolytes & IV Therapy
Fluids, Electrolytes & IV TherapyFluids, Electrolytes & IV Therapy
Fluids, Electrolytes & IV Therapy
 
FLUID THERAPY IN NURSING
FLUID THERAPY IN NURSINGFLUID THERAPY IN NURSING
FLUID THERAPY IN NURSING
 
Pressure Ulcer Prevention & Skin Care
Pressure Ulcer Prevention & Skin CarePressure Ulcer Prevention & Skin Care
Pressure Ulcer Prevention & Skin Care
 

Similar to FLUIDS AND ELECTROLYTES

Fluid electrolyte balance Adult health .pptx
Fluid electrolyte balance Adult health .pptxFluid electrolyte balance Adult health .pptx
Fluid electrolyte balance Adult health .pptx
nimram374
 
Medication and fluid therapy.pptx
Medication and fluid therapy.pptxMedication and fluid therapy.pptx
Medication and fluid therapy.pptx
MohammedAbdela7
 
Fluid and Electrolytes.pptx
Fluid and Electrolytes.pptxFluid and Electrolytes.pptx
Fluid and Electrolytes.pptx
abdiasis omar mohamed
 
fluid imbalance and its management.pptx
fluid imbalance and its management.pptxfluid imbalance and its management.pptx
fluid imbalance and its management.pptx
Sapana Shrestha
 
fluidbalance (1).ppt
fluidbalance (1).pptfluidbalance (1).ppt
fluidbalance (1).ppt
UzairMangrio
 
Fluids &amp; electrolytes seminar [autosaved]
Fluids &amp; electrolytes seminar [autosaved]Fluids &amp; electrolytes seminar [autosaved]
Fluids &amp; electrolytes seminar [autosaved]
Avinash Rathore
 
vnd.openxmlformats-officedocument.presentationml.presentation&rendition=1-7.pptx
vnd.openxmlformats-officedocument.presentationml.presentation&rendition=1-7.pptxvnd.openxmlformats-officedocument.presentationml.presentation&rendition=1-7.pptx
vnd.openxmlformats-officedocument.presentationml.presentation&rendition=1-7.pptx
SureshPharamasivam
 
FLUID & ELECTROLYTE IMBALANCE
FLUID & ELECTROLYTE IMBALANCEFLUID & ELECTROLYTE IMBALANCE
FLUID & ELECTROLYTE IMBALANCE
Flavia Dass
 
Diabetes insipidus
Diabetes insipidusDiabetes insipidus
Diabetes insipidus
Bob Kiyemba
 
Fluid and Electrolytes
Fluid and ElectrolytesFluid and Electrolytes
Fluid and Electrolytes
Eric Pazziuagan
 
Fluid Therapy in Companion Animals
Fluid Therapy in Companion AnimalsFluid Therapy in Companion Animals
Fluid Therapy in Companion Animals
Veterinary Doctor
 
Fluids and Electrolytes
Fluids and ElectrolytesFluids and Electrolytes
Fluids and ElectrolytesTosca Torres
 
Fluid and electrolyte balance
Fluid and electrolyte balanceFluid and electrolyte balance
Fluid and electrolyte balanceShermil Sayd
 
Fluid & electrolyte imbalance
Fluid & electrolyte imbalanceFluid & electrolyte imbalance
Fluid & electrolyte imbalance
Assistant Professor
 
fluid-electrolyte-imbalance-n132-160210135651.pdf
fluid-electrolyte-imbalance-n132-160210135651.pdffluid-electrolyte-imbalance-n132-160210135651.pdf
fluid-electrolyte-imbalance-n132-160210135651.pdf
versha26
 
Fluid &amp; electrolyte imbalance
Fluid &amp; electrolyte imbalanceFluid &amp; electrolyte imbalance
Fluid &amp; electrolyte imbalance
Harish Kumawat
 
Fluidsandelectrolytes
FluidsandelectrolytesFluidsandelectrolytes
Fluidsandelectrolytes
shenell delfin
 
Fluid management-Dehydration-Hypovolemia.pptx
Fluid management-Dehydration-Hypovolemia.pptxFluid management-Dehydration-Hypovolemia.pptx
Fluid management-Dehydration-Hypovolemia.pptx
MatinMahmudov
 
Water and electrolyte balance 2022 .pptx
Water and electrolyte balance 2022 .pptxWater and electrolyte balance 2022 .pptx
Water and electrolyte balance 2022 .pptx
munda492
 
Fluid and Electrolyte
Fluid and ElectrolyteFluid and Electrolyte
Fluid and Electrolyte
BhartiAntil
 

Similar to FLUIDS AND ELECTROLYTES (20)

Fluid electrolyte balance Adult health .pptx
Fluid electrolyte balance Adult health .pptxFluid electrolyte balance Adult health .pptx
Fluid electrolyte balance Adult health .pptx
 
Medication and fluid therapy.pptx
Medication and fluid therapy.pptxMedication and fluid therapy.pptx
Medication and fluid therapy.pptx
 
Fluid and Electrolytes.pptx
Fluid and Electrolytes.pptxFluid and Electrolytes.pptx
Fluid and Electrolytes.pptx
 
fluid imbalance and its management.pptx
fluid imbalance and its management.pptxfluid imbalance and its management.pptx
fluid imbalance and its management.pptx
 
fluidbalance (1).ppt
fluidbalance (1).pptfluidbalance (1).ppt
fluidbalance (1).ppt
 
Fluids &amp; electrolytes seminar [autosaved]
Fluids &amp; electrolytes seminar [autosaved]Fluids &amp; electrolytes seminar [autosaved]
Fluids &amp; electrolytes seminar [autosaved]
 
vnd.openxmlformats-officedocument.presentationml.presentation&rendition=1-7.pptx
vnd.openxmlformats-officedocument.presentationml.presentation&rendition=1-7.pptxvnd.openxmlformats-officedocument.presentationml.presentation&rendition=1-7.pptx
vnd.openxmlformats-officedocument.presentationml.presentation&rendition=1-7.pptx
 
FLUID & ELECTROLYTE IMBALANCE
FLUID & ELECTROLYTE IMBALANCEFLUID & ELECTROLYTE IMBALANCE
FLUID & ELECTROLYTE IMBALANCE
 
Diabetes insipidus
Diabetes insipidusDiabetes insipidus
Diabetes insipidus
 
Fluid and Electrolytes
Fluid and ElectrolytesFluid and Electrolytes
Fluid and Electrolytes
 
Fluid Therapy in Companion Animals
Fluid Therapy in Companion AnimalsFluid Therapy in Companion Animals
Fluid Therapy in Companion Animals
 
Fluids and Electrolytes
Fluids and ElectrolytesFluids and Electrolytes
Fluids and Electrolytes
 
Fluid and electrolyte balance
Fluid and electrolyte balanceFluid and electrolyte balance
Fluid and electrolyte balance
 
Fluid & electrolyte imbalance
Fluid & electrolyte imbalanceFluid & electrolyte imbalance
Fluid & electrolyte imbalance
 
fluid-electrolyte-imbalance-n132-160210135651.pdf
fluid-electrolyte-imbalance-n132-160210135651.pdffluid-electrolyte-imbalance-n132-160210135651.pdf
fluid-electrolyte-imbalance-n132-160210135651.pdf
 
Fluid &amp; electrolyte imbalance
Fluid &amp; electrolyte imbalanceFluid &amp; electrolyte imbalance
Fluid &amp; electrolyte imbalance
 
Fluidsandelectrolytes
FluidsandelectrolytesFluidsandelectrolytes
Fluidsandelectrolytes
 
Fluid management-Dehydration-Hypovolemia.pptx
Fluid management-Dehydration-Hypovolemia.pptxFluid management-Dehydration-Hypovolemia.pptx
Fluid management-Dehydration-Hypovolemia.pptx
 
Water and electrolyte balance 2022 .pptx
Water and electrolyte balance 2022 .pptxWater and electrolyte balance 2022 .pptx
Water and electrolyte balance 2022 .pptx
 
Fluid and Electrolyte
Fluid and ElectrolyteFluid and Electrolyte
Fluid and Electrolyte
 

More from Rommel Luis III Israel

Nursing Care of Client With Acute And Chronic Renal Failure.ppt
Nursing Care of Client With Acute And Chronic Renal Failure.pptNursing Care of Client With Acute And Chronic Renal Failure.ppt
Nursing Care of Client With Acute And Chronic Renal Failure.ppt
Rommel Luis III Israel
 
ABDOMINAL COMPARTMENT SYSNDROME
ABDOMINAL COMPARTMENT SYSNDROMEABDOMINAL COMPARTMENT SYSNDROME
ABDOMINAL COMPARTMENT SYSNDROME
Rommel Luis III Israel
 
DOH PROGRAMS RELATED TO FAMILY PLANNING.pptx
DOH PROGRAMS RELATED TO FAMILY PLANNING.pptxDOH PROGRAMS RELATED TO FAMILY PLANNING.pptx
DOH PROGRAMS RELATED TO FAMILY PLANNING.pptx
Rommel Luis III Israel
 
ORTHOPEDIC NURSING: CARE OF THE CLIENT WITH MUSCULO-SKELETAL DISORDER
ORTHOPEDIC NURSING: CARE OF THE CLIENT WITH MUSCULO-SKELETAL DISORDERORTHOPEDIC NURSING: CARE OF THE CLIENT WITH MUSCULO-SKELETAL DISORDER
ORTHOPEDIC NURSING: CARE OF THE CLIENT WITH MUSCULO-SKELETAL DISORDER
Rommel Luis III Israel
 
LABORATORY PROCEDURES-ALTERATION IN THE ENDOCRINE SYSTEM
LABORATORY PROCEDURES-ALTERATION IN THE ENDOCRINE SYSTEMLABORATORY PROCEDURES-ALTERATION IN THE ENDOCRINE SYSTEM
LABORATORY PROCEDURES-ALTERATION IN THE ENDOCRINE SYSTEM
Rommel Luis III Israel
 
THE END OF LIFE (EOL): CARE OF THE CLIENT
THE END OF LIFE (EOL): CARE OF THE CLIENTTHE END OF LIFE (EOL): CARE OF THE CLIENT
THE END OF LIFE (EOL): CARE OF THE CLIENT
Rommel Luis III Israel
 
CIRRHOSIS OF THE LIVER, LIVER CANCER and HEPATITIS
CIRRHOSIS OF THE LIVER, LIVER CANCER and HEPATITISCIRRHOSIS OF THE LIVER, LIVER CANCER and HEPATITIS
CIRRHOSIS OF THE LIVER, LIVER CANCER and HEPATITIS
Rommel Luis III Israel
 
Nursing Diagnosis: The Second Phase of The Nursing Process
Nursing Diagnosis: The Second Phase of The Nursing ProcessNursing Diagnosis: The Second Phase of The Nursing Process
Nursing Diagnosis: The Second Phase of The Nursing Process
Rommel Luis III Israel
 
CARE OF THE CLIENTS WITH ACUTE AND CHRONIC RENAL FAILURE
CARE OF THE CLIENTS WITH ACUTE AND CHRONIC RENAL FAILURECARE OF THE CLIENTS WITH ACUTE AND CHRONIC RENAL FAILURE
CARE OF THE CLIENTS WITH ACUTE AND CHRONIC RENAL FAILURE
Rommel Luis III Israel
 
Care of Client With Liver and Pancreatic Disorder
Care of Client With  Liver and Pancreatic DisorderCare of Client With  Liver and Pancreatic Disorder
Care of Client With Liver and Pancreatic Disorder
Rommel Luis III Israel
 
The Evidence-Based Practice in Nursing (EBP)
The Evidence-Based Practice in Nursing  (EBP)The Evidence-Based Practice in Nursing  (EBP)
The Evidence-Based Practice in Nursing (EBP)
Rommel Luis III Israel
 
NATIONAL IMMUNIZATION PROGRAM Formerly EPI)
NATIONAL IMMUNIZATION PROGRAM Formerly EPI)NATIONAL IMMUNIZATION PROGRAM Formerly EPI)
NATIONAL IMMUNIZATION PROGRAM Formerly EPI)
Rommel Luis III Israel
 
SCOPE OF CRITICAL CARE ORGANIZATION
SCOPE OF CRITICAL CARE ORGANIZATIONSCOPE OF CRITICAL CARE ORGANIZATION
SCOPE OF CRITICAL CARE ORGANIZATION
Rommel Luis III Israel
 
MEDICATION ADMINISTRATION AND BASIC COMPUTATION
MEDICATION ADMINISTRATION AND BASIC COMPUTATIONMEDICATION ADMINISTRATION AND BASIC COMPUTATION
MEDICATION ADMINISTRATION AND BASIC COMPUTATION
Rommel Luis III Israel
 
ENDOCRINE SYSTEM: REVIEW OF THE PATHOPHYSIOLOGY
ENDOCRINE SYSTEM: REVIEW OF THE PATHOPHYSIOLOGYENDOCRINE SYSTEM: REVIEW OF THE PATHOPHYSIOLOGY
ENDOCRINE SYSTEM: REVIEW OF THE PATHOPHYSIOLOGY
Rommel Luis III Israel
 
ENDOCRINE SYSTEM - ANATOMY AND PHYSIOLOGY
ENDOCRINE SYSTEM - ANATOMY AND PHYSIOLOGYENDOCRINE SYSTEM - ANATOMY AND PHYSIOLOGY
ENDOCRINE SYSTEM - ANATOMY AND PHYSIOLOGY
Rommel Luis III Israel
 
BREATHING AND INCENTIVE SPIROMETRY PRESENTATION
BREATHING AND INCENTIVE SPIROMETRY PRESENTATIONBREATHING AND INCENTIVE SPIROMETRY PRESENTATION
BREATHING AND INCENTIVE SPIROMETRY PRESENTATION
Rommel Luis III Israel
 
Theory of Culture Care Diversity and Universality
Theory of Culture Care Diversity and UniversalityTheory of Culture Care Diversity and Universality
Theory of Culture Care Diversity and Universality
Rommel Luis III Israel
 
CARE OF THE CLIENT WITH RESPIRATORY DISORDER.ppt
CARE OF THE CLIENT WITH RESPIRATORY DISORDER.pptCARE OF THE CLIENT WITH RESPIRATORY DISORDER.ppt
CARE OF THE CLIENT WITH RESPIRATORY DISORDER.ppt
Rommel Luis III Israel
 
ADVANCE CARDIAC LIFE SUPPORT - (ACLS vs. BLS)
ADVANCE CARDIAC LIFE SUPPORT -  (ACLS vs. BLS)ADVANCE CARDIAC LIFE SUPPORT -  (ACLS vs. BLS)
ADVANCE CARDIAC LIFE SUPPORT - (ACLS vs. BLS)
Rommel Luis III Israel
 

More from Rommel Luis III Israel (20)

Nursing Care of Client With Acute And Chronic Renal Failure.ppt
Nursing Care of Client With Acute And Chronic Renal Failure.pptNursing Care of Client With Acute And Chronic Renal Failure.ppt
Nursing Care of Client With Acute And Chronic Renal Failure.ppt
 
ABDOMINAL COMPARTMENT SYSNDROME
ABDOMINAL COMPARTMENT SYSNDROMEABDOMINAL COMPARTMENT SYSNDROME
ABDOMINAL COMPARTMENT SYSNDROME
 
DOH PROGRAMS RELATED TO FAMILY PLANNING.pptx
DOH PROGRAMS RELATED TO FAMILY PLANNING.pptxDOH PROGRAMS RELATED TO FAMILY PLANNING.pptx
DOH PROGRAMS RELATED TO FAMILY PLANNING.pptx
 
ORTHOPEDIC NURSING: CARE OF THE CLIENT WITH MUSCULO-SKELETAL DISORDER
ORTHOPEDIC NURSING: CARE OF THE CLIENT WITH MUSCULO-SKELETAL DISORDERORTHOPEDIC NURSING: CARE OF THE CLIENT WITH MUSCULO-SKELETAL DISORDER
ORTHOPEDIC NURSING: CARE OF THE CLIENT WITH MUSCULO-SKELETAL DISORDER
 
LABORATORY PROCEDURES-ALTERATION IN THE ENDOCRINE SYSTEM
LABORATORY PROCEDURES-ALTERATION IN THE ENDOCRINE SYSTEMLABORATORY PROCEDURES-ALTERATION IN THE ENDOCRINE SYSTEM
LABORATORY PROCEDURES-ALTERATION IN THE ENDOCRINE SYSTEM
 
THE END OF LIFE (EOL): CARE OF THE CLIENT
THE END OF LIFE (EOL): CARE OF THE CLIENTTHE END OF LIFE (EOL): CARE OF THE CLIENT
THE END OF LIFE (EOL): CARE OF THE CLIENT
 
CIRRHOSIS OF THE LIVER, LIVER CANCER and HEPATITIS
CIRRHOSIS OF THE LIVER, LIVER CANCER and HEPATITISCIRRHOSIS OF THE LIVER, LIVER CANCER and HEPATITIS
CIRRHOSIS OF THE LIVER, LIVER CANCER and HEPATITIS
 
Nursing Diagnosis: The Second Phase of The Nursing Process
Nursing Diagnosis: The Second Phase of The Nursing ProcessNursing Diagnosis: The Second Phase of The Nursing Process
Nursing Diagnosis: The Second Phase of The Nursing Process
 
CARE OF THE CLIENTS WITH ACUTE AND CHRONIC RENAL FAILURE
CARE OF THE CLIENTS WITH ACUTE AND CHRONIC RENAL FAILURECARE OF THE CLIENTS WITH ACUTE AND CHRONIC RENAL FAILURE
CARE OF THE CLIENTS WITH ACUTE AND CHRONIC RENAL FAILURE
 
Care of Client With Liver and Pancreatic Disorder
Care of Client With  Liver and Pancreatic DisorderCare of Client With  Liver and Pancreatic Disorder
Care of Client With Liver and Pancreatic Disorder
 
The Evidence-Based Practice in Nursing (EBP)
The Evidence-Based Practice in Nursing  (EBP)The Evidence-Based Practice in Nursing  (EBP)
The Evidence-Based Practice in Nursing (EBP)
 
NATIONAL IMMUNIZATION PROGRAM Formerly EPI)
NATIONAL IMMUNIZATION PROGRAM Formerly EPI)NATIONAL IMMUNIZATION PROGRAM Formerly EPI)
NATIONAL IMMUNIZATION PROGRAM Formerly EPI)
 
SCOPE OF CRITICAL CARE ORGANIZATION
SCOPE OF CRITICAL CARE ORGANIZATIONSCOPE OF CRITICAL CARE ORGANIZATION
SCOPE OF CRITICAL CARE ORGANIZATION
 
MEDICATION ADMINISTRATION AND BASIC COMPUTATION
MEDICATION ADMINISTRATION AND BASIC COMPUTATIONMEDICATION ADMINISTRATION AND BASIC COMPUTATION
MEDICATION ADMINISTRATION AND BASIC COMPUTATION
 
ENDOCRINE SYSTEM: REVIEW OF THE PATHOPHYSIOLOGY
ENDOCRINE SYSTEM: REVIEW OF THE PATHOPHYSIOLOGYENDOCRINE SYSTEM: REVIEW OF THE PATHOPHYSIOLOGY
ENDOCRINE SYSTEM: REVIEW OF THE PATHOPHYSIOLOGY
 
ENDOCRINE SYSTEM - ANATOMY AND PHYSIOLOGY
ENDOCRINE SYSTEM - ANATOMY AND PHYSIOLOGYENDOCRINE SYSTEM - ANATOMY AND PHYSIOLOGY
ENDOCRINE SYSTEM - ANATOMY AND PHYSIOLOGY
 
BREATHING AND INCENTIVE SPIROMETRY PRESENTATION
BREATHING AND INCENTIVE SPIROMETRY PRESENTATIONBREATHING AND INCENTIVE SPIROMETRY PRESENTATION
BREATHING AND INCENTIVE SPIROMETRY PRESENTATION
 
Theory of Culture Care Diversity and Universality
Theory of Culture Care Diversity and UniversalityTheory of Culture Care Diversity and Universality
Theory of Culture Care Diversity and Universality
 
CARE OF THE CLIENT WITH RESPIRATORY DISORDER.ppt
CARE OF THE CLIENT WITH RESPIRATORY DISORDER.pptCARE OF THE CLIENT WITH RESPIRATORY DISORDER.ppt
CARE OF THE CLIENT WITH RESPIRATORY DISORDER.ppt
 
ADVANCE CARDIAC LIFE SUPPORT - (ACLS vs. BLS)
ADVANCE CARDIAC LIFE SUPPORT -  (ACLS vs. BLS)ADVANCE CARDIAC LIFE SUPPORT -  (ACLS vs. BLS)
ADVANCE CARDIAC LIFE SUPPORT - (ACLS vs. BLS)
 

Recently uploaded

Telehealth Psychology Building Trust with Clients.pptx
Telehealth Psychology Building Trust with Clients.pptxTelehealth Psychology Building Trust with Clients.pptx
Telehealth Psychology Building Trust with Clients.pptx
The Harvest Clinic
 
Dimensions of Healthcare Quality
Dimensions of Healthcare QualityDimensions of Healthcare Quality
Dimensions of Healthcare Quality
Naeemshahzad51
 
CANCER CANCER CANCER CANCER CANCER CANCER
CANCER  CANCER  CANCER  CANCER  CANCER CANCERCANCER  CANCER  CANCER  CANCER  CANCER CANCER
CANCER CANCER CANCER CANCER CANCER CANCER
KRISTELLEGAMBOA2
 
Essential Metrics for Palliative Care Management
Essential Metrics for Palliative Care ManagementEssential Metrics for Palliative Care Management
Essential Metrics for Palliative Care Management
Care Coordinations
 
Artificial Intelligence to Optimize Cardiovascular Therapy
Artificial Intelligence to Optimize Cardiovascular TherapyArtificial Intelligence to Optimize Cardiovascular Therapy
Artificial Intelligence to Optimize Cardiovascular Therapy
Iris Thiele Isip-Tan
 
Global launch of the Healthy Ageing and Prevention Index 2nd wave – alongside...
Global launch of the Healthy Ageing and Prevention Index 2nd wave – alongside...Global launch of the Healthy Ageing and Prevention Index 2nd wave – alongside...
Global launch of the Healthy Ageing and Prevention Index 2nd wave – alongside...
ILC- UK
 
HEAT WAVE presented by priya bhojwani..pptx
HEAT WAVE presented by priya bhojwani..pptxHEAT WAVE presented by priya bhojwani..pptx
HEAT WAVE presented by priya bhojwani..pptx
priyabhojwani1200
 
A Community health , health for prisoners
A Community health  , health for prisonersA Community health  , health for prisoners
A Community health , health for prisoners
Ahmed Elmi
 
India Clinical Trials Market: Industry Size and Growth Trends [2030] Analyzed...
India Clinical Trials Market: Industry Size and Growth Trends [2030] Analyzed...India Clinical Trials Market: Industry Size and Growth Trends [2030] Analyzed...
India Clinical Trials Market: Industry Size and Growth Trends [2030] Analyzed...
Kumar Satyam
 
Neuro Saphirex Cranial Brochure
Neuro Saphirex Cranial BrochureNeuro Saphirex Cranial Brochure
Neuro Saphirex Cranial Brochure
RXOOM Healthcare Pvt. Ltd. ​
 
Antibiotic Stewardship by Anushri Srivastava.pptx
Antibiotic Stewardship by Anushri Srivastava.pptxAntibiotic Stewardship by Anushri Srivastava.pptx
Antibiotic Stewardship by Anushri Srivastava.pptx
AnushriSrivastav
 
ICH Guidelines for Pharmacovigilance.pdf
ICH Guidelines for Pharmacovigilance.pdfICH Guidelines for Pharmacovigilance.pdf
ICH Guidelines for Pharmacovigilance.pdf
NEHA GUPTA
 
Deep Leg Vein Thrombosis (DVT): Meaning, Causes, Symptoms, Treatment, and Mor...
Deep Leg Vein Thrombosis (DVT): Meaning, Causes, Symptoms, Treatment, and Mor...Deep Leg Vein Thrombosis (DVT): Meaning, Causes, Symptoms, Treatment, and Mor...
Deep Leg Vein Thrombosis (DVT): Meaning, Causes, Symptoms, Treatment, and Mor...
The Lifesciences Magazine
 
GURGAON Call Girls ❤8901183002❤ #ℂALL# #gIRLS# In GURGAON ₹,2500 Cash Payment...
GURGAON Call Girls ❤8901183002❤ #ℂALL# #gIRLS# In GURGAON ₹,2500 Cash Payment...GURGAON Call Girls ❤8901183002❤ #ℂALL# #gIRLS# In GURGAON ₹,2500 Cash Payment...
GURGAON Call Girls ❤8901183002❤ #ℂALL# #gIRLS# In GURGAON ₹,2500 Cash Payment...
ranishasharma67
 
Health Education on prevention of hypertension
Health Education on prevention of hypertensionHealth Education on prevention of hypertension
Health Education on prevention of hypertension
Radhika kulvi
 
Myopia Management & Control Strategies.pptx
Myopia Management & Control Strategies.pptxMyopia Management & Control Strategies.pptx
Myopia Management & Control Strategies.pptx
RitonDeb1
 
POLYCYSTIC OVARIAN SYNDROME (PCOS)......
POLYCYSTIC OVARIAN SYNDROME (PCOS)......POLYCYSTIC OVARIAN SYNDROME (PCOS)......
POLYCYSTIC OVARIAN SYNDROME (PCOS)......
Ameena Kadar
 
VVIP Dehradun Girls 9719300533 Heat-bake { Dehradun } Genteel ℂall Serviℂe By...
VVIP Dehradun Girls 9719300533 Heat-bake { Dehradun } Genteel ℂall Serviℂe By...VVIP Dehradun Girls 9719300533 Heat-bake { Dehradun } Genteel ℂall Serviℂe By...
VVIP Dehradun Girls 9719300533 Heat-bake { Dehradun } Genteel ℂall Serviℂe By...
rajkumar669520
 
Haridwar ❤CALL Girls 🔝 89011★83002 🔝 ❤ℂall Girls IN Haridwar ESCORT SERVICE❤
Haridwar ❤CALL Girls 🔝 89011★83002 🔝 ❤ℂall Girls IN Haridwar ESCORT SERVICE❤Haridwar ❤CALL Girls 🔝 89011★83002 🔝 ❤ℂall Girls IN Haridwar ESCORT SERVICE❤
Haridwar ❤CALL Girls 🔝 89011★83002 🔝 ❤ℂall Girls IN Haridwar ESCORT SERVICE❤
ranishasharma67
 
CHAPTER 1 SEMESTER V - ROLE OF PEADIATRIC NURSE.pdf
CHAPTER 1 SEMESTER V - ROLE OF PEADIATRIC NURSE.pdfCHAPTER 1 SEMESTER V - ROLE OF PEADIATRIC NURSE.pdf
CHAPTER 1 SEMESTER V - ROLE OF PEADIATRIC NURSE.pdf
Sachin Sharma
 

Recently uploaded (20)

Telehealth Psychology Building Trust with Clients.pptx
Telehealth Psychology Building Trust with Clients.pptxTelehealth Psychology Building Trust with Clients.pptx
Telehealth Psychology Building Trust with Clients.pptx
 
Dimensions of Healthcare Quality
Dimensions of Healthcare QualityDimensions of Healthcare Quality
Dimensions of Healthcare Quality
 
CANCER CANCER CANCER CANCER CANCER CANCER
CANCER  CANCER  CANCER  CANCER  CANCER CANCERCANCER  CANCER  CANCER  CANCER  CANCER CANCER
CANCER CANCER CANCER CANCER CANCER CANCER
 
Essential Metrics for Palliative Care Management
Essential Metrics for Palliative Care ManagementEssential Metrics for Palliative Care Management
Essential Metrics for Palliative Care Management
 
Artificial Intelligence to Optimize Cardiovascular Therapy
Artificial Intelligence to Optimize Cardiovascular TherapyArtificial Intelligence to Optimize Cardiovascular Therapy
Artificial Intelligence to Optimize Cardiovascular Therapy
 
Global launch of the Healthy Ageing and Prevention Index 2nd wave – alongside...
Global launch of the Healthy Ageing and Prevention Index 2nd wave – alongside...Global launch of the Healthy Ageing and Prevention Index 2nd wave – alongside...
Global launch of the Healthy Ageing and Prevention Index 2nd wave – alongside...
 
HEAT WAVE presented by priya bhojwani..pptx
HEAT WAVE presented by priya bhojwani..pptxHEAT WAVE presented by priya bhojwani..pptx
HEAT WAVE presented by priya bhojwani..pptx
 
A Community health , health for prisoners
A Community health  , health for prisonersA Community health  , health for prisoners
A Community health , health for prisoners
 
India Clinical Trials Market: Industry Size and Growth Trends [2030] Analyzed...
India Clinical Trials Market: Industry Size and Growth Trends [2030] Analyzed...India Clinical Trials Market: Industry Size and Growth Trends [2030] Analyzed...
India Clinical Trials Market: Industry Size and Growth Trends [2030] Analyzed...
 
Neuro Saphirex Cranial Brochure
Neuro Saphirex Cranial BrochureNeuro Saphirex Cranial Brochure
Neuro Saphirex Cranial Brochure
 
Antibiotic Stewardship by Anushri Srivastava.pptx
Antibiotic Stewardship by Anushri Srivastava.pptxAntibiotic Stewardship by Anushri Srivastava.pptx
Antibiotic Stewardship by Anushri Srivastava.pptx
 
ICH Guidelines for Pharmacovigilance.pdf
ICH Guidelines for Pharmacovigilance.pdfICH Guidelines for Pharmacovigilance.pdf
ICH Guidelines for Pharmacovigilance.pdf
 
Deep Leg Vein Thrombosis (DVT): Meaning, Causes, Symptoms, Treatment, and Mor...
Deep Leg Vein Thrombosis (DVT): Meaning, Causes, Symptoms, Treatment, and Mor...Deep Leg Vein Thrombosis (DVT): Meaning, Causes, Symptoms, Treatment, and Mor...
Deep Leg Vein Thrombosis (DVT): Meaning, Causes, Symptoms, Treatment, and Mor...
 
GURGAON Call Girls ❤8901183002❤ #ℂALL# #gIRLS# In GURGAON ₹,2500 Cash Payment...
GURGAON Call Girls ❤8901183002❤ #ℂALL# #gIRLS# In GURGAON ₹,2500 Cash Payment...GURGAON Call Girls ❤8901183002❤ #ℂALL# #gIRLS# In GURGAON ₹,2500 Cash Payment...
GURGAON Call Girls ❤8901183002❤ #ℂALL# #gIRLS# In GURGAON ₹,2500 Cash Payment...
 
Health Education on prevention of hypertension
Health Education on prevention of hypertensionHealth Education on prevention of hypertension
Health Education on prevention of hypertension
 
Myopia Management & Control Strategies.pptx
Myopia Management & Control Strategies.pptxMyopia Management & Control Strategies.pptx
Myopia Management & Control Strategies.pptx
 
POLYCYSTIC OVARIAN SYNDROME (PCOS)......
POLYCYSTIC OVARIAN SYNDROME (PCOS)......POLYCYSTIC OVARIAN SYNDROME (PCOS)......
POLYCYSTIC OVARIAN SYNDROME (PCOS)......
 
VVIP Dehradun Girls 9719300533 Heat-bake { Dehradun } Genteel ℂall Serviℂe By...
VVIP Dehradun Girls 9719300533 Heat-bake { Dehradun } Genteel ℂall Serviℂe By...VVIP Dehradun Girls 9719300533 Heat-bake { Dehradun } Genteel ℂall Serviℂe By...
VVIP Dehradun Girls 9719300533 Heat-bake { Dehradun } Genteel ℂall Serviℂe By...
 
Haridwar ❤CALL Girls 🔝 89011★83002 🔝 ❤ℂall Girls IN Haridwar ESCORT SERVICE❤
Haridwar ❤CALL Girls 🔝 89011★83002 🔝 ❤ℂall Girls IN Haridwar ESCORT SERVICE❤Haridwar ❤CALL Girls 🔝 89011★83002 🔝 ❤ℂall Girls IN Haridwar ESCORT SERVICE❤
Haridwar ❤CALL Girls 🔝 89011★83002 🔝 ❤ℂall Girls IN Haridwar ESCORT SERVICE❤
 
CHAPTER 1 SEMESTER V - ROLE OF PEADIATRIC NURSE.pdf
CHAPTER 1 SEMESTER V - ROLE OF PEADIATRIC NURSE.pdfCHAPTER 1 SEMESTER V - ROLE OF PEADIATRIC NURSE.pdf
CHAPTER 1 SEMESTER V - ROLE OF PEADIATRIC NURSE.pdf
 

FLUIDS AND ELECTROLYTES

  • 3. HOW IMPORTANT IS WATER? • Between 50% and 60% of the human body by weight is water • Water provides a medium for transporting nutrients to cells and wastes from cells and for transporting substances such as hormones, enzymes, blood platelets, and red and white blood cells • Water facilitates cellular metabolism and proper cellular chemical functioning • Water acts as a solvent for electrolytes and nonelectrolytes • Helps maintain normal body temperature • Facilitates digestion and promotes elimination • Acts as a tissue lubricant
  • 4. VARIATIONS IN FLUID CONTENT BODY FAT Because fat cells contain little water and lean tissue is rich in water, the more obese the person, the smaller the percentage of total body water compared with body weight. This is also true between sexes because females tend to have proportionally more body fat than males. There is also an increase in fat cells in older people
  • 5. VARIATIONS IN FLUID CONTENT AGE
  • 6.
  • 8. ↓Blood volume or ↓BP Volume receptor Atria and great veins Hypothalamus ↓ Posterior pituitary gland Osmoreceptors in hypothalamus ↑Osmolarity ↑ADH Kidney tubules ↑H2O reabsorption ↑vascular volume and ↓osmolarity Narcotics, Stress, Anesthetic agents, Heat, Nicotine, Antineoplastic agents, Surgery ANTIDIURETIC HORMONE REGULATION MECHANISMS
  • 9. Juxtaglomerular cells-kidney ↓Serum Sodium ↓Blood volume Angiotensin I Kidney tubules Angiotensin II Adrenal Cortex ↑Sodium resorption (H2O resorbed with sodium); ↑ Blood volume Angiotensinogen in plasma RENIN Angiotensin- converting enzyme ALDOSTERONE Intestine, sweat glands, Salivary glands Via vasoconstriction of arterial smooth muscle ALDOSTERONE-RENIN-ANGIOTENSIN SYSTEM
  • 10. Fluid Types • Fluids in the body generally aren’t found in pure forms • Isotonic, hypotonic, and hypertonic types • Defined in terms of the amount of solute or dissolve substances in the solution • Balancing these fluids involves the shifting of fluid not the solute involved
  • 11. Isotonic Solutions • No net fluid shifts occur between isotonic solutions because the solution are equally concentrated • Ex. NSS or 0.9SS
  • 12. Hypotonic Solutions • Has a lower solute concentration than another solution • Fluid from the hypotonic solution would shift into the second solution until the two solutions had equal concentrations • Ex. Half normal or 0.45%SS
  • 14. Fluid Movements • Fluids and solutes constantly move within the body, which allows the body to maintain homeostasis • Fluids along with nutrients and waste products constantly shift within the body’s compartments from the cell to the interstitial spaces, to the blood vessels and back again
  • 17. LABORATORY TESTS FOR EVALUATING FLUID STATUS • • • • • •
  • 19. FLUID BALANCE • The desirable amount of fluid intake and loss in adults ranges from 1500 to 3500 mL each 24 hours. Ave= 2500 mL • Normally INTAKE = OUTPUT FLUID IMBALANCE • Changes in ECF volume = alterations in sodium balance • Change in sodium/water ratio = either hypoosmolarity or hyperosmolarity • Fluid excess or deficit = loss of fluid balance • As with all clinical problems, the same pathophysiologic change is not of equal significance to all people • For example, consider two persons who have the same viral syndrome with associated nausea and vomiting
  • 20. FLUID DEFICIT/HYPOVOLEMIA • May occur as a result of: – Reduced fluid intake – Loss of body fluids – Sequestration (compartmentalizing) of body fluids Pathophysiology and Clinical Manifestations DECREASED FLUID VOLUME Stimulation of thirst center in hypothalamus Person complains of thirst ↑ ADH Secretion ↑ Water resorption ↓ Urine Output Renin-Angiotensin- Aldosterone System Activation ↑ Sodium and Water Resorption ↑ Urine specific gravity
  • 21. Pathophysiology and Clinical Manifestations UNTREATED FLUID VOLUME DEFICIT Depletion of fluids available ↑ BODY TEMPERATURE Dry mucous membranes Difficulty with speech Cells become unable to continue providing water to replace ECF losses Signs of circulatory collapse ↓ blood pressure ↑ heart rate ↑ respiratory rate Restlessness and Apprehension
  • 22. Hypovolemia • Nursing Intervention • Monitor fluid intake and output • Checked daily weight (a 1lb(0.45kg) weight loss equals a 500 ml fluid loss) • Monitor hemodynamic values such as CVP • Monitor results of laboratory studies • Assess level of consciousness • Administer and monitor I.V. fluids • Apply and adjust oxygen therapy as ordered • If patient is bleeding, apply direct continuous pressure to the area and elevate it if possible • Assess skin turgor • Assess oral mucous membranes • Turn the patient at least every 2 hours to prevent skin breakdown • Encourage oral fluids
  • 24. Collaborative Care Management Identification of vulnerable patients and risk factors: * Compromised mental state * Physical limitations * Disease states * Limited access to adequate food and fluids Development of a plan of care Family members should be educated about the importance of fluid and nutrition intake Collaboration with the nurse, patient, family members, and other health care providers for continued assessment and treatment of problems Ongoing assessment and detailed action plan of fluid and serum electrolyte balance. Factors such as medications (particularly diuretics), hyperventilation, fever, burns, diarrhea, and diabetes with appropriate referral
  • 30.
  • 31. Composition of Different Intravenous Solution IVF Dextrose (g/L) Na (meq/L) Cl (meq/L) K (meq/L) Lactate (meq/L) D5 0.9% NaCl 50 154 154 D5 0.15% NaCl 50 25 25 D5 0.3% NaCl 50 51 51 D5 0.45% NaCl 50 77 77 D5 IMB 50 25 22 20 23 LRS 0 130 109 4 28 NSS 0 154 154 D5LRS 50 130 109 4 28 31
  • 32. Fluid Replacement Therapy ISOTONIC SOLUTION Facts Examples Uses -same osmolality as plasma (app. 275 to 295 mOsm/kg) -vascular space osmolality not altered by infusion -expand intracellular and extracellular space equally; degree of expansion correlates with amount of fluid infused -no solution-related shifting between ICF and ECF spaces -cells neither shrink nor swell with fluid movement Dextrose 5% in water, Normal Saline Solution, Lactated Ringers Solution -Fluid loss and dehydration -Hypernatremia -Blood transfusion, fluid challenges, resuscitation, shock, metabolic alkalosis, hypercalcemia, hyponatremia -Acute blood loss, burns, dehydration, hypovolemia 32
  • 35. FLUID EXCESS/HYPERVOLEMIA Psychiatric Disorders, SIADH, Certain head injuries Dietary Sodium Indiscretion Renal and endocrine disturbances, malignancies, adenomas Overhydration Excessive Sodium Intake Failure of renal or hormonal regulatory functions FLUID VOLUME EXCESS/HYPERVOLEMIA
  • 36. • Since ECF becomes hypoosmolar, fluid moves into the cells to equalize the concentration on both sides of the cell membrane • Thus there, is an increase in intracellular fluid • The brain cells are particularly sensitive to the increase of intracellular water, the most common signs of hypoosmolar overhydration are changes in mental status. Confusion, ataxia, and convulsions may also occur. • Other clinical manifestations include: hyperventilation, sudden weight gain, warm, moist skin, increased ICP: slow bounding pulse with an increase in systolic and decrease in diastolic pressue and peripheral edema, usually not marked
  • 37. Hypervolemia • Evaluating pitting edema • Press your fingertip firmly into the patients skin over a bony surface for a few seconds. Then note the depth of the imprint your finger leaves on the skin • A slight imprint indicates +1 pitting edema • A deep imprint, with the skin slow to return to its original contour, indicates a +4 pitting edema • When the skin resists pressure and appears distended, the condition is called brawny edema, which causes the skin to swell so much that fluid cant be displaced
  • 38. Hypervolemia • Diagnostic Findings: • Decreased hematocrit resulting from hemodilution • Normal serum Na level • Low serum K and BUN levels • either due to hemodilution or higher levels may indicate renal failure • Low oxygen level • Abnormal chest x-ray • Indicates fluid accumulation • May reveal pulmonary edema or pleural effusions
  • 39. Hypervolemia • Treatment • Na and fluid intake restriction • Diuretics to promote excess fluid excretion • Morphine and nitroglycerin (Nitro-Dur) for pulmonary edema • Dilate blood vessels • Reduce pulmonary congestion and amount of blood returning to the heart • Digoxin for heart failure • Strengthens cardiac contractions
  • 40. Hypervolemia • Treatment • Supportive measures • Oxygen administration • Bed rest • Hemodialysis or continuous renal replacement therapy for renal dysfunction
  • 41. Hypervolemia • Nursing Interventions • Monitor fluid intake and output • Monitor daily weight • Monitor cardiopulmonary status • Auscultate breathe sounds • Assess for complaints of dyspnea • Monitor chest x-ray results • Monitor arterial blood gas values • Assess for peripheral edema • Inspect the patient for sacral edema • Monitor infusion of I.V. solutions • Monitor the effects of prescribed medications
  • 42. General Information • Involve destruction of the epidermis, dermis, or subcutaneous layers of the skin • Can be permanently disfiguring and incapacitating and possibly life- threatening
  • 43. General Information • Associated imbalances result from alterations in skin integrity and internal body membranes, and from effect of heat on body water and solute loss that may result from cellular destruction
  • 44. General Information •Type and severity of imbalance depends on burn type and depth, percentage body surface area involved and burn phase
  • 45. Pathophysiology •Burn Phase: –Refer to stages that describe physiologic changes occurring after a burn Burn phase Fluid- accumulation phase Fluid- remobilization phase Convalescent phase
  • 46. Pathophysiology Fluid-accumulation phase:  Last fro 36 to 48 hours after a burn injury  Fluid shifts from vascular compartment to interstitial space – third-space shift  Edema caused by shifted fluid, which typically reaches maximum within 8 hours after injury  Circulation possibly compromised and pulses diminished from severe edema Burn phase Fluid- accumulation phase Fluid- remobilization phase Convalescent phase
  • 47. Pathophysiology • Several reasons for fluid imbalances during fluid- accumulation phase –Damage to capillaries causing altered vessel permeability –Diminished kidney perfusion –Production and release of stress hormones such as aldosterone and ADH Burn phase Fluid- accumulation phase Fluid- remobilization phase Convalescent phase
  • 48. Pathophysiology Respiratory problems Muscle and tissue injuries GI problems Electrolyte imbalances:  Common during fluid accumulation phase due to body’s hypermetabolic needs and priority that fluid replacement takes over nutritional needs during emergency phase Burn phase Fluid- accumulation phase Fluid- remobilization phase Convalescent phase
  • 49. Pathophysiology Fluid- remobilization phase :  Also known as diuresis stage  Starts about 48 hours after initial burn  Fluid shifted back to vascular compartment  Edema at burn site decreased, blood flow to kidneys increased, increased urine output  Fluid and electrolyte imbalances can still occur Burn phase Fluid- accumulation phase Fluid- remobilizatio n phase Convalescent phase
  • 50. Pathophysiology Convalescent phase:  Begins after first two phases has been resolved  Characterized by healing or reconstruction of burn wound  Major fluid shifts now resolved but possible further fluid and electrolyte imbalances exist as a result of inadequate dietary intake  Anemia is common – severe burns typically destroy red blood cells Burn phase Fluid- accumulation phase Fluid- remobilization phase Convalescent phase
  • 51. Characteristics • 1. Minor Burns • Partial thickness burns are no greater than 15% of the TBSA in the adult • Full thickness burns are < 2% of the TBSA in the adult • Burn areas do not involve the eyes, ears, hands, face, feet, or perineum • There are no electrical burns or inhalation injuries • The client is an adult younger than 60 y.o. • The client has no preexisting medical condition at the time of the burn injury • No other injury occurred with the burn
  • 52. Characteristics 2. Moderate Burns a. Partial thickness burns are deep and are 15% to 25% of the TBSA in the adult b. Full thickness burns are 2% to 10% of the TBSA in the adult c. Burn areas do not involve the eyes, ears, hands, face, feet, or perineum d. There are no electrical burns or inhalation injuries e. The client is an adult younger than 60 y.o. f. The client has no chronic cardiac, pulmonary, or endocrine disorder at the time of the burn injury g. No other complicated injury occurred with the burn
  • 53. Characteristics 3. Major Burns a. Partial thickness burns are > 25% of the TBSA in the adult b. Full thickness burns are > 10% of the TBSA c. Burn areas involve the eyes, ears, hands, face, feet, or perineum d. The burn injury was an electrical or inhalation injury e. The client is older than 60 y.o. f. The client has a chronic cardiac, pulmonary, or metabolic disorder at the time of the burn injury g. Burns are accompanied by other injuries
  • 54. 54 Burn:Classification Superficial (1°burns) • Involve only the epidermal layer of the skin. • sunburns are commonly first- degree burns.
  • 57. 57 • Present of blisters indicates superficial partial-thickness injury. • Blister may ↑size because continuous exudation and collection of tissue fluid. • Healing phase of partial thickness, itching and dryness because ↑vascularization of sebaceous glands, ↓reduction of secretions and ↑perspiration. Partial thickness (2°burn)
  • 60. 60 Burn:Classification 3.Full thickness (third-degree burn) • Destruction of the epidermis and the entire dermis, subcutaneous layer, muscle and bone. • Nerve ending are destroyed-painless wound. • Eschar may be formed due to surface dehydration. • Black networks of coagulate capillaries may be seen. • Need skin grafting because the destroyed tissue is unable to epithelialize. • Deep partial-thickness burn may convert to a full-thickness burn because of infection, trauma or ↓blood supply.
  • 64. 64 Extent of surface area burned Rule of nines-An estimated of the TBSA involved as a result of a burn. The rule of nines measures the percentage of the body burned by dividing the body into multiples of nine. The initial evaluation is made upon arrival at the hospital.
  • 65. 65 Lund and Browder • More precise method of estimating • Recognizes that the percentage of BSA of various anatomic parts. • By dividing the body into very small areas and providing an estimate of proportion of BSA accounted for by such body parts • Includes, a table indicating the adjustment for different ages • Head and trunk represent larger proportions of body surface in children.
  • 66. 66
  • 67. 67 Age in years 0 1 5 10 15 Adult A-head (back or front) 9½ 8 ½ 6½ 5½ 4½ 3½ B-1 thigh (back or front) 2¾ 3 ¼ 4 4¼ 4½ 4¾ C-1 leg (back or front) 2½ 2 ½ 2¾ 3 3¼ 3½ Lund and Browder chart
  • 68. TYPES OF BURNS Thermal Burns: caused by exposure to flames, hot liquids, steam or hot objects Chemical Burns: Caused by tissue contact with strong alkali, or organic compounds Systemic toxicity from cutaneous absorption can occur Radiation Burns: caused by exposure to UV light, x-rays, or radioactive source
  • 69. TYPES OF BURNS • Electrical Burns: • Caused by heat generated by electrical energy as it passes through the body • Results in internal tissue damage • Cutaneous burns cause muscle and soft tissue damage that may be extensive, particularly in high voltage electrical injuries • Alternating current is more dangerous than direct current because it is associated with CP arrest, ventricular fibrillation, tetanic muscle contractions, and long bone or vertebral fractures
  • 70. Potential Imbalance • Hypovolemia • Approximately 10% of plasma volume lost into tissue soon after a severe burn • Occurs because of the third space shift causes multiple effects: • With burn’s damage to the skin surface, decrease in skins ability to prevent water loss; patient can lose up to 8L of fluid per day (400ml/hour) • Potential for blood loss, adding to fluid volume losses
  • 71. Potential Imbalance • Hypervolemia • Usually develops 3 to 5 days after a major burn injury • Occurs during the fluid remobilization phase, as fluid shifts from the interstitial space back to the vascular compartment • May be exacerbated by excessive administration of I.V. fluids
  • 73. Burns NURSING PRIORITY: The client with burn injury is often awake, mentally alert, and cooperative at first. The level of consciousness may change as respiratory status change or as the fluid shift occurs, precipitating hypovolemia. If the client is unconscious or confused, assess him or her for the possibility of a head injury.
  • 74. Burns • Assess for – Patent airway – Presence of adequate breath sounds – Symptoms of hypoxia – Pulmonary damage • Burns around the face, neck, mouth or in the oral mucosal area – Circulatory status • Tachycardia and hypotension occur early • Elevate UO
  • 75. Burns • Assess for – GI function – check last time client ate – Fluid status • UO (30 ml/hr) • Hypotension (< 90/60) • Confusion / disorientation – Circulatory status of the extremities
  • 76. Burns Treatment  Respiratory status takes priority over the treatment of the burn injury  If burn area is small  cold compress or immerse in cool water (not ice) to ↓ heat  May have ointment on the burn area  Analgesics IV, IM, SQ. oral forms may not be absorbed effectively
  • 77. Burns • Nursing intervention – Maintain patent airway; prevent hypoxia – Evaluate fluid status; determine circulatory status – Prevent of decrease infection – Maintain nutrition – Prevent contractures and scarring – Promote acceptance and adaptation to alterations in body image
  • 78. Burns Formula name Electrolyte- Containing solution Colloid-Containing Solution Dextrose in Water Evans NSS 1 ml/kg/%burn NSS 1 ml/kg/%burn 2000 ml Brooke LR 1.5 ml/kg/%burn 0.5 ml/kg/%burn 2000 ml Modified Brooke LR 2 ml/kg/%burn None None Parkland LR 4 ml/kg/%burn None None Hypertonic Saline Fluid containing 250 mEq of Na/L to maintain hourly urine output of 70 ml in adults None None First 24 hours
  • 79. Burns Formula name Electrolyte- Containing solution Colloid-Containing Solution Dextrose in Water Evans ½ of first 24-hr requirement ½ of first 24-hr requirement 2000 ml Brooke ½ - ¾ of first 24-hr requirement ½ - ¾ of first 24-hr requirement 2000 ml Modified Brooke None 0.3-0.5 ml/kg/%burn Titrate to maintain urine output Parkland None 0.3-0.5 ml/kg/%burn Titrate to maintain urine output Hypertonic Saline Same solution to maintain hourly urine output of 30 ml in adults None None Second 24 hours
  • 80. Considerations AGE AND GENERAL HEALTH Mortality rates are higher for children < 4 y.o, particularly those < 1 y.o., and for clients over the age of 60 years. Debilitating disorders, such as cardiac, respiratory, endocrine, and renal d/o, negatively influence the client’s response to injury and treatment. Mortality rate is higher when the client has a pre-existing disorder at the time of the burn injury
  • 82. Which one is not a cation? A. Calcium B. Magnesium C. Phosphorous D. Sodium
  • 83. Anions and Cations • Anions • Cations Bicarbonate Chloride Phosphorous Calcium Magnesium Potassium Sodium
  • 84.
  • 86. Controls and regulates volume of body fluids Its concentration is the major determinant of ECF volume Is the chief electrolyte of ECF Influence ICF Volume Participates in the generation and transmission of nerve impulses Is an essential electrolyte in the sodium- potassium pump RDA: not known precisely. 500 mg Eliminated primarily by the kidneys, smaller in feces and perspiration Salt intake affects sodium concentrations Sodium is conserved through reabsorption in the kidneys, a process stimulated by aldosterone Normal value: 135-145 mEq/L 86
  • 87. HYPONAT REMIA Refers to the serum sodium concentration less than 135 mEq/L Common with thiazide diuretic use, but may also be seen with loop and potassium-sparing diuretics as well Occurs with marked sodium restriction, vomiting and diarrhea, SIADH, etc. The etiology may be mulfactorial May also occur postop due to temporary alteration in hypothalamic function, loss of GI fluids by vomiting or suction, or hydration with nonelectrolyte solutions Postoperative hyponatremia is a more serious complication in premenopausal women. The reasons behind this is unknown Therefore monitoring serum levels is critical and careful assessment for symptoms of hyponatremia is important for all postoperative patients
  • 88. PATHOPHYSIOLOGY OF HYPONATREMIA Sodium loss from the intravascular compartment Diffusion of water into the interstitial spaces Sodium in the interstitial space is diluted Decreased osmolarity of ECF Water moves into the cell as a result of sodium loss Extracellular compartment is depleted of water CLINICAL SYMPTOMS
  • 89. CLINICAL MANIFESTATIONS OF HYPONATREMIA Muscle Weakness APATHY Postural hypotension Nausea and Abdominal Cramps Weight Loss In severe hyponatremia: mental confusion, delirium, shock and coma
  • 90. COLLABORATIVE CARE MANAGEMENT • General goal: correct sodium imbalance and restore normal fluid and electrolyte homeostasis • Recognition of people at risk for hyponatremia is essential for its prevention: athletes, persons working in hot environments • Salt is always replaced along with water • Management includes educating vulnerable people to recognize signs and symptoms of sodium depletion and maintaining sufficient sodium and water intake to replace skin and insensible fluid loss • Generally, an increased sodium and water intake provides adequate treatment • Education as the importance of sodium and fluid balance and the rationale for prescription medications to ensure compliance • Daily weight. MIO • Monitoring of sodium levels to determine extent of replacement • Generally, PNSS or PLRS is prescribed • Too rapid restoration of sodium balance, hypertonic sodium solutions may provoke brain injury
  • 91. HYPERNATREMIA • A serum sodium level above 145 mEq/L is termed hypernatremia • May occur as a result of fluid deficit or sodium excess • Frequently occurs with fluid imbalance • Develops when an excess of sodium occurs without a proportional increase in body fluid or when water loss occurs without proportional loss of sodium • Risk Factors: excess dietary or parenteral sodium intake, watery diarrhea, diabetes insipidus, damage to thirst center, those with physical or mental status compromise, and people with hypothalamic dysfunction
  • 92. PATHOPHYSIOLOGY OF HYPERNATREMIA Increased Sodium concentration in ECF Osmolarity rises Water leaves the cell by osmosis and enters the the extracellular compartments Dilution of fluids in ECF Cells are water depleted Suppression of aldosterone secretion Sodium is exreted in the urine CLINICAL SYMPTOMS
  • 93. CLINICAL MANIFESTATIONS Dry, sticky mucous membranes Firm, rubbery tissue turgor Manic excitement Tachycardia DEATH
  • 94. COLLABORATIVE CARE MANAGEMENT Recognition of risk factors: bedridden and debilitated patients, diabetes insipidus, fluid deprivation, the elderly and the very young A careful and accurate record of MIO permits quick recognition of negative fluid balance People with kidney failure, CHF, or increased aldosterone production may require dietary sodium intake restriction Usually, osmolar balance can be restored with oral fluids. If not, the parenteral route may be necessary Fluid resuscitation must be undertaken with particular caution in patients with compromised cardiac or renal function The nurse should closely monitor the patient’s response to fluids and be alert to symptoms of fluid overload
  • 95. • Major cation of the ICF. Chief regulator of cellular enzyme activity and cellular water content • The more K, the less Na. The less K, the more Na • Plays a vital role in such processes such as transmission of electrical impulses, particularly in nerve, heart, skeletal, intestinal and lung tissue; CHON and CHO metabolism; and cellular building; and maintenance of cellular metabolism and excitation • Assists in regulation of acid-base balance by cellular exchange with H • RDA: not known precisely. 50-100 mEq • Sources: bananas, peaches, kiwi, figs, dates, apricots, oranges, prunes, melons, raisins, broccoli, and potatoes, meat, dairy products • Excreted primarily by the kidneys. No effective conserving mechanism • Conserved by sodium pump and kidneys when levels are low • Aldosterone triggers K excretion in urine • Normal value: 3.5 – 5 mEq/L 95
  • 96. CAUSES AND EFFECTS OF HYPOKALEMIA • Known as a low level of serum potassium, less than 3.5 mEq/L Decreased Intake ↓ Food and Fluids as in starvation Failure to replace GI losses Increased Loss ↑ Aldosterone Gastrointestinal losses Potassium-losing diuretics Loss from cells as in trauma, burns Shift of Potassium into Cells (No change in total body potassium) HYPOKALEMIA GI Tract Anorexia N&V Abdominal distention CNS Lethargy, Diminished deep-tendon reflexes, Confusion, Mental depression Muscles Weakness, Flaccid paralysis, Weakness of respiratory muscles, Respiratory arrest CV System Decrease in standing BP, Dysrhythmias, ECG changes, Myocardial damage, Cardiac arrest Kidneys ↓Capacity to concentrate waste, water loss, thirst, kidney damage
  • 97. PATHOPHYSIOLOGY OF HYPOKALEMIA = Action Potential Nerve and Muscle Activity Low Extracellular K+ Increase in resting membrane potential The cell becomes less excitable
  • 98. Aldosterone is secreted Sodium is retained in the body through resorption by the kidney tubules Potassium is excreted Use of certain diuretics such as thiazides and furosemide, and corticosteroids Increased urinary output Loss of potassium in urine
  • 99. COLLABORATIVE CARE MANAGEMENT • Being alert to the conditions that cause potassium depletion such as vomiting, diarrhea and diuretics, by monitoring the patient for early warning signs • No more than 3 enemas without consulting a physician • Education about the importance of adequate dietary intake of potassium • In severe hypokalemia, a patient may die unless potassium is administered promptly • The safest way to administer K is orally. When K is given IV, the rate of flow must be monitored closely and should be diluted. Should not exceed 20 mEq/hr • If PO, taken with at least ½ glass of water • Cardiac monitoring is useful • Potassium sparing diuretics such as triamterene, spironolactone, etc • Symptoms of K depletion: muscle weakness, anorexia, nausea and vomiting = appropriate referral
  • 100. CAUSES AND EFFECTS OF HYPERKALEMIA • Serum potassium level greater than 5.5 mEq/L Excess Intake Dietary intake of excess of kidney’s ability to excrete; Excess parenteral administration Decreased Loss Potassium-sparing diuretics; Renal failure; Adrenal insufficiency Shift of Potassium out of the Cells Extensive injuries, crushing injuries, metabolic acidosis HYPERKALEMIA GI Tract N&V Diarrhea, Colic CNS Numbness, paresthesias Muscles Early: irritability Late: weakness leading to flaccid paralysis CV System Conduction disturbance, ventricular fibrillation, Cardiac Arrest Kidneys Oliguria leading to anuria
  • 101. COLLABORATIVE CARE MANAGEMENT • Patients at risk should be identified: impaired renal function to avoid OTC, esp. NSAIDS which provoke hyperkalemia; and salt substitutes that are high in potassium • Severity guides therapy – Mild: Withholding provoking agent (i.e., K supp) – Severe (>6 mEq/L: cation-exchange resin such as Kayexalate (act by exchanging the cations in the resin for the potassium in the intestine  potassium is then excreted in the stool; Continuous cardiac monitoring • Bowel function must be maintained if Kayexelate therapy is to be effective • Potassium-wasting diuretics may be prescribed to promote further potassium loss. Dialysis for patients with renal failure to eliminate excess potassium • Intravenous Ca Gluconate may be prescribed to counteract the cardiac effects of hyperkalemia • Insulin infusions and IV NaCO3 may be used to promote intracellular uptake of K
  • 102. • Most abundant electrolyte in the body. 99% in bones and teeth • Close link between calcium and phosphorus. High PO4, Low Ca • Necessary for nerve impulse transmission and blood clotting and is also a catalyst for muscle contraction and other cellular activities • Needed for Vitamin B12 absorption and use • Necessary for strong bones and teeth and thickness and strength of cell membranes • RDA: 1g for adults. Higher for children and pregnant and lactating women according to body weight, older people, esp. post-menopausal • Found in milk, cheese, and dried beans; some in meat and vegetables • Use is stimulated by Vitamin D. Excreted in urine, feces, bile, digestive secretions, and perspiration • Normal value 8.5 – 10.5 mg/dl 102
  • 103. CAUSES AND EFFECTS OF HYPOCALCEMIA Decreased Ionized Ca Large tranfusion with citrated blood Excess Loss Kidney Disease Decrease in GI Tract and Bone Absorption ↑Magnesium ↑Calcitonin ↓Vitamin D ↓Parathyroid Hormone HYPOCALCEMIA Bones Osteoporosis leading to Fractures CNS Tingling ↓ convulsions Other Abnormal deposits of calcium in body tissues Muscles Muscle spasm ↓ Tetany Cardiovascular System Dysrhythmias ↓ Cardiac arrest Inadequate Intake Dietary Deficit
  • 104. PATHOPHYSIOLOGY OF HYPOCALCEMIA •Calcium ions are thought to line the pores of cell membranes, especially neurons •Calcium and Sodium repel each other •When serum calcium levels are low, this blocking effect is minimized •When Sodium moves more easily into the cell, depolarization takes place more easily •This results in increased excitability of the nervous system leading to muscle spasm, tingling sensations, and if severe, convulsions and tetany •Skeletal, smooth, and cardiac muscle functions are all affected by overstimulation Sodium Calcium
  • 105. CLINICAL MANIFESTATIONS OF HYPOCALCEMIA COMPLAINT OF NUMBNESS AND TINGLING OF EARS, NOSE, FINGERTIPS OR TOES TREATMENT PAINFUL MUSCULAR SPASMS (TETANY) ESPECIALLY OF FEET AND HANDS (CARPOPEDAL SPASMS), MUSCLE TWITCHING AND CONVULSIONS MAY FOLLOW
  • 106. TESTS USED TO ELICIT SIGNS OF CALCIUM DEFICIENCY
  • 107. COLLABORATIVE CARE MANAGEMENT • Identify risk factors: Inadequate calcium intake, excess calcium loss, Vitamin D deficiency, patients with poor diets • Education about the importance of adequate calcium and Vitamin D intake • Patients undergoing thyroid, parathyroid, and radical neck surgery are particularly vulnerable to hypocalcemia secondary to parathyroid hormone deficit • Monitoring of serum calcium levels and correction of deficits • Citrate is added to store blood to prevent coagulation. • Citrate + Transfusion = Citrate+Calcium • Normally, Liver + Citrate = Quick metabolism • Preexisting calcium deficit/hepatic dysfunction/large amounts of BT very rapidly = hypocalcemia • With acute hypocalcemia, Ca Gluconate is used + Continuous cardiac monitoring • Mild Hypocalcemia: High calcium diet or oral calcium salts • If PTH or Vit D Deficiency is the cause: aluminum hydroxide gel is used because when serum phosphate level rises, calcium level falls • Complication: Bone demineralization • Therefore, careful ambulation should be encouraged to minimize bone resorption
  • 108. HYPERCALCEMIA: Serum concentration > 10mg/dL Causes and Effects Loss from bones Immobilization, Carcinoma with bone metastases, Multiple myeloma Excess Intake ↑ Calcium diet (esp. milk) Antacids containing calcium Increase in factors Causing Mobilization from bone ↑PTH, ↑ Vitamin D, steroid therapy HYPERCALCEMIA Kidneys Stones ↓ Kidney Damage CNS ↓Deep-tendon reflexes ↓ Lethargy ↓ Coma Bones Bone pain ↓ Osteoporosis ↓ Fractures Muscles Muscle fatigue, hypotonia ↓ ↓ GI motility CV System Depressed activity ↓ Dysrhythmias ↓ Cardiac Arrest
  • 109. HOW IT HAPPENS HYPERCALCEMIA DEPRESSED NERVE AND MUSCLE ACTIVITY DEEP TENDON REFLEXES MAY BE DECREASED OR ABSENT MYOCARDIAL FUNCTION IS ALTERED
  • 110. CLINICAL MANIFESTATIONS OF HYPERCALCEMIA Decreased GI Motility Cardiac Dysrhythmias Constipation Nausea Mental status changes: lethargy, confusion, memory loss
  • 111. CLINICAL MANIFESTATIONS OF HYPERCALCEMIA Immobilization Bone Demineralization Calcium accumulates in the ECF and passes through the kidneys Ca Precipitation Calcium Stones
  • 112. COLLABORATIVE CARE MANAGEMENT • Mild hypercalcemia: hydration and education about avoiding foods high in calcium or medications that promote calcium elevation • Ambulation as appropriate; weight-bearing exercises as tolerated • Trapeze, resistance devices • Marked hypercalcemia: prevention of pathologic fractures, individualized plan of care • Prevention of renal calculi: encourage oral fluids to prevent concentrated urine: 3000 to 4000 mL/day unless contraindicated • Acid-ash fruit juices: cranberry juice and prune juice • Severe hypercalcemia: medical emergency: continuous cardiac monitoring, hydration, IV furosemide, Calcitonin and/or plicamycin (mithramycin), q2 serum and urinary electrolytes
  • 113. • Mostly found within body cells: heart, bone, nerve, and muscle tissues • Second most important cation in the ICF, 2nd to K+ • Functions: Metabolism of CHO and CHON, protein and DNA synthesis, DNA and RNA transcription, and translation of RNA, maintains normal intracellular levels of potassium, helps maintain electric activity in nervous tissue membranes and muscle membranes • RDA: about 18-30 mEq; children require larger amounts • Sources: vegetables, nuts, fish, whole grains, peas, and beans • Absorbed in the intestines and excreted by the kidneys • Plasma concentrations of magnesium range from 1.5 – 2.5 mEq/L, with about one third of that amount bound to plasma proteins 113
  • 114. HYPOMAGNESEMIA: Serum level < 1.5 mEq/L • Usually coexists with hypokalemia and less often with hypocalcemia Decreased Intake Prolonged malnutrition, Starvation Impaired absorption from GI Tract Malabsorption syndrome, Alcohol Withdrawal Syndrome, Hypercalcemia, Diarrhea, Draining gastrointestinal fistula Excessive Excretion ↑Aldosterone, Conditions causing large losses of urine HYPOMAGNESEMIA Mental Changes Agitation, Depression, Confusion CNS Convulsions, Paresthesias, Tremor, Ataxia Muscles Cramps, Spasticity, Tetany CV System Tachycardia, Hypotension, Dysrhythmias HYPOKALEMIA
  • 115. PATHOPHYSIOLOGY OF HYPOMAGNESEMIA Low serum magnesium level Increased acetylcholine release Increased neuromuscular irritability Increased sensitivity to acetylcholine at the myoneural junction Diminished threshold of excitation for the motor nerve Enhancement of myofibril contraction
  • 116. PATHOPHYSIOLOGY OF HYPOMAGNESEMIA High Serum Calcium Increased acetylcholine release Increased neuromuscular irritability Increased sensitivity to acetylcholine at the myoneural junction Diminished threshold of excitation for the motor nerve Enhancement of myofibril contraction High Serum Calcium Excretion of Magnesium By the GI tract
  • 117. PATHOPHYSIOLOGY OF HYPOMAGNESEMIA MAGNESIUM INHIBITS TRANSPORT OF PTH DECREASE IN THE AMOUNT OF CALCIUM BEING RELEASED FROM THE BONE POSSIBLE CALCIUM DEFICIT
  • 118. CLINICAL MANIFESTATIONS OF HYPOMAGNESEMIA CONFUSION DEPRESSION CRAMPS TETANY CONVULSIONS
  • 119. COLLABORATIVE CARE MANAGEMENT • Recognition of people at risk: people taking loop diuretics and digoxin should be encouraged to eat foods rich in magnesium, such as fruits, vegetables, cereals, and milk • Recognition of signs and symptoms of magnesium deficiency • Magnesium is essential for potassium resorption, so if hypokalemia does not respond to potassium replacement, hypomagnesemia should be suspected • Treatment of the underlying cause is the first consideration in hypomagnesemia • Severe: parenteral magnesium replacement is indicated • IV therapy: continuous cardiac monitoring • Safety measures for patients with mental status changes
  • 120. HYPER MAGNE SEMIA: Serum Mg level 2.5 mEq/L • Seldom develops in the presence of normal renal function • May occur as a result of Mg replacement • May occur when MgSO4 is administered to prevent seizures resulting from eclampsia • Careful monitoring is imperative
  • 121. PATHOPHYSIOLOGY Renal failure, Excessive IV infusion of magnesium, Decreased GI elimination and/or absorption, etc. Accummulation of Mg in the body Diminishing of reflexes, drowsiness, lethargy Mg Level Rises Severe Respiratory Depression RESPIRATORY ARREST may occur Altered Electrical Conduction Slowed heart rate and AV Block Peripheral vasodilation Hypotension, flushing, and increased skin warmth
  • 122. COLLABORATIVE CARE MANAGEMENT • Identification of patients at risk: those with impaired renal function to avoid OTC that contain magnesium such as Milk of Magnesia and some Mg-containing antacids • Any patient receiving parenteral magnesium therapy should be assessed frequently for signs of hypermagnesemia • Mild hypermagnesemia: withholding magnesium-containing medications may suffice • Renal failure: dialysis • Severe: may require treatment with calcium gluconate (10-20 mL of 10% Ca Gluconate administered over 10 minutes) • If cardiorespiratory collapse is imminent, the patient may require temporary pacemaker and ventilator support
  • 124. Parameter_____Fluid Excess___ Fluid Loss/Electrolyte Imbalance____ Behavior Tires easily; Change in behavior, confusion, apathy Head, neck Facial edema, distended neck Headache, thirst, dry mucous membranes veins Upper GI Anorexia, nausea, vomiting Skin Warm, moist, taut, cool feeling Dry, decreased turgor where edematous Respiration Dyspnea, orthopnea, productive Changes in rate and depth of respiration cough, moist breath sounds Circulation Loss of sensation in edematous Pulse rate changes, dysrhythmia, postural areas, pallor, bounding pulse, increased blood pressure hypotension Abdomen Increased girth, fluid wave Distention, abdominal cramps Elimination Constipation Diarrhea, constipation Extremities Dependent edema, “pitting” Muscle weakness, tingling, tetany , discomfort from weight of bedclothes
  • 126. LABORATORY VALUES FLUID DEFICIT FLUID EXCESS Hemoconcentration Hemodilution ↑ Hct, BUN, E+ levels ↓ Hct, BUN, E+ levels ↑ Urine Specific Gravity ↓ Urine Specific Gravity
  • 127. Determined from analysis of patient data Diagnostic Title Possible Etiologic Factors 1 Deficient fluid volume Active fluid volume loss (hemorrhage, diarrhea, gastric intubation, wounds, diaphoresis), inadequate fluid intake, failure of regulatory mechanisms, sequestration of body fluids 2 Excess Fluid Volume Excess fluid intake, excess sodium intake, compromised regulatory processes
  • 128. EXPECTED PATIENT OUTCOMES 1. Will maintain functional fluid volume as evidenced by adequate urinary output, stable weight, normal vital signs, normal urine specific gravity, moist mucus membranes, balanced intake and output, elastic skin turgor, prompt capillary refill, and absence of edema 2. Will verbalize understanding of treatment plan and causative factors that led to the imbalance
  • 129. 1,2Intake and Output Monitoring - Type and amount of fluid the patient has received and the route by which they were administered - Record of solid food intake. Gelatin or Popsicles are recorded as fluids - Ice chips are recorded by dividing the amount of chips by ½ (60 mL of chips = 30 mL water) - Accurate output record and described by color, content, and odor (Normally, gastric contents are watery and pale yellow-green; they usually have a sour odor) - With acid-base balance upset, gastric secretions may have a fruity odor because of ketone bodies - Bile: thicker than gastric juice, dark green to brown, acrid odor, bitter taste when vomiting - NGT irrigation added to intake - Stools: difficult to estimate amount; consistency, color, and number of stools provide a reasonable estimate - Peritoneal or pleural fluid drainage is recorded as output as with its amount, color, and clarity - Character and volume of urine. Place signs and materials so that an accurate record of UO is maintained
  • 130. 1,2 Intake and Output Monitoring - Evaluate and refer urine specific gravity as appropriate (normal value is 1.003 – 1.030). The implications are: High Dehydration Low SIADH, overhydration - Drainage, fluid aspirated from any body cavity must be measured. With dressings, fluid loss is the difference between the wet dressings and the dry weight of the dressing - Accurate recording of the temperature to help the physician determine how much fluid should be replaced 1,2 Daily Weight - Evaluate trends in weight (An increase in 1kg in weight is equal to the retention of 1L of fluid in an edematous patient) Considerations: - Daily weights early in the morning after voiding but before he or she has eaten or defecated
  • 131. 1 Replacement of Fluid and Electrolytes General Principles: - Either by oral intake (healthiest way), tube feeding, intravenous infusion, and/or total parenteral nutrition - Normal saline solution and plain water should also be given by slow drip to replace daily fluid loss - IV administration per doctor’s orders - Fluid replacement considerations: * Most effective when apportioned over 24 hr period (Better regulation, ↓potential for calculi formation and subsequent renal damage, ↓potential for circulatory overload which may cause in fluid and electrolyte shifts) * Administer concentrated solutions of Na, Glucose or protein because they require body fluids for dilution * Consider the size of the patient (small adult has less fluid in each compartment, especially in the intravascular compartment) - Promote oral intake as appropriate * Caution with coffee, tea, and some colas
  • 132. * small amount at frequent intervals is more useful than a large amount presented less often * Always give consideration to cultural and aesthetic aspects of eating - Give mouth care to a dehydrated patient before and after meals and before bedtime (Xerostomia may lead to disruption of t issues in the oral cavity) - Avoid irritating foods - Stimulation of saliva may be aided by hard candy or chewing gum or carboxymethylcellulose (artificial saliva) - Keep lips moist and well lubricated - Give salty broth or soda crackers for sodium replacement and tea or orange juice for potassium replacement as appropriate. Bananas, citrus fruits and juices, some fresh vegetables, coffee, and tea are relatively high in potassium and low in sodium. Milk, meat, eggs, and nuts are high in protein, sodium and potassium. - Offer milk for patients with draining fistulas from any portion of the GI tract. Lactose intolerance is not necessarily a contraindication (Lactase enzyme preparations are available) - Increase usual daily requirement of foods when losses must be restored, as tolerated
  • 133. * Patients with cardiac and renal impairments are instructed to avoid foods containing high levels of sodium, potassium and bicarbonate - Administer replacement solutions through tube feeding as is * Either water, physiologic solution of NaCl, high protein liquids, or a regular diet can be blended, diluted and given by gavage * The water content in the tube feeding needs to be increased if: 1 the patient complains of thirst 2 the protein or electrolyte content of the tube feeding is high 3 the patient has fever or disease causing an increased metabolic rate 4 UO is concentrated 5 signs of water deficit develop - Administer parenteral fluids as necessary
  • 134. * Types of solutions - D5W (hypotonic) is given short-term for hyponatremia - D5NSS may be given depending on the serum levels of sodium and vascular volume + KCl to meet normal intake needs and replace losses for hyponatremia - Dextrose 5% in 0.2% normal saline is generally used as a maintenance fluid - Dextrose 5% in ½ normal saline is generally used as a replacement solution for losses caused by gastrointestinal drainage - PNSS is given primarily when large amounts of sodium have been lost and for patients with hyponatremia - LRS is also isotonic because it remains in the extracellular space - Fructose or 10-20% glucose in distilled water are hypertonic solutions and may partially meet body needs for CHOs - Dextran (commonly-used plasma expander) increases plasma volume by increasing oncotic pressure. May cause prolonged bleeding time and is CI in patients with renal failure, bleeding disorders, or severe CHF
  • 135. * Administration - The rate should be regulated according to the patient’s needs and condition per doctor’s orders - Monitor UO carefully. Refer marked decreases! - Verify orders for potassium administration in patients with renal failure and untreated adrenal insufficiency - Usual rate for fluid loss replacement: 3ml/min - Recognize signs of pulmonary edema (bounding pulse, engorged peripheral veins, hoarseness, dyspnea, cough, and rales) that can result from ↑IV rate - If infiltration occurs, the infusion should be stopped immediately and relocated. Peripheral IV sites are generally rotated every 72 hours - For dextran and other plasma expanders, observe for anaphylactic reaction (apprehension, dyspnea, wheezing, tightness of chest, angioedema, itching, hives and hypotension). If this happens, switch infusion to nonprotein solution and run at KVO rate, notify physician and monitor VS - Pronounced and continued thirst despite administration of fluids is not normal and should be reported (may indicate DM or hypercalcemia)
  • 136. * Patient/Family Education - Include the signs and symptoms of water excess in discharge instructions - With drug therapy, instruct patient and family regarding correct method of administration, correct dose, and therapeutic and adverse effects - Instruct to read labels for nutritional content * For K restriction: avoid organ meats, fresh and dried fruits, and salt substitutes - Skin assessment and care, positioning techniques for patients with mobility restrictions
  • 137. * Achievement of outcomes is successful in disturbances in fluid and electrolyte balance: 1 Maintains functional fluid volume level with adequate UO, VS within the patient’s normal limits, sp gr of urine within 1.003-1.035, moist mucous membranes, stable weight, Intake=output, elastic skin turgor, and no edema 2 States possible causes of imbalance and plan to prevent recurrence of imbalances 3 Reports a decrease or absence of symptoms causing discomfort
  • 139. DRAWING ARTERIAL BLOOD GASES ALLEN’S TEST ARTERIAL PUNCTURE
  • 140.
  • 141. BASIC REGULATION OF ACID-BASE BALANCE CO2 + H2O ↔ H2CO3 ↔ H+ + HCO3 The lungs help control acid-base balance by blowing off or retaining CO2. The kidneys help regulate acid-base balance by excreting or retaining HCO3
  • 142. TYPES OF ACID-BASE DISTURBANCES Depression of the central nervous system, as evidenced by disorientation followed by coma Overexcitability of the nervous system; muscles may go into a state of tetany and convulsioons
  • 143.
  • 144.
  • 146. RESPIRATORY ACIDOSIS: CARBONIC ACID EXCESS Damage to the respiratory center in the medulla, drug or narcotic use, obstruction of respiratory passages, respiratory and respiratory muscle disorders Decrease in the rate of pulmonary ventilation Increase in the concentration of CO2, carbonic acid, and hydrogen ions RESPIRATORY ACIDOSIS Potassium moves out of the cells HYPERKALEMIA VENTRICULAR FIBRILLATION
  • 147.
  • 148. NURSING MANAGEMENT OF RESPIRATORY ACIDOSIS ASSESSMENT * Health Hx: complaints of headache, confusion, lethargy, nausea, irritability, nausea, irritability, anxiety, dyspnea, and blurred vision, preexisting conditions * Physical Examination: lethargy to stupor to coma, tachycardia, hypertension, cardiac dysrhythmias, airway patency NURSING DIAGNOSES include but are not limited to: • Diagnostic Title Possible Etiologic Factors 1 Impaired gas exchange Hypoventilation 2 Disturbed thought processes Central nervous system depression 3 Anxiety Hypoxia, hospitalization 4 Risk for ineffective family Illness of a family member coping 5 Ineffective airway clearance Hypoventilation, secretions 6 Ineffective breathing pattern Hypoventilation, dyspnea
  • 149. NURSING MANAGEMENT OF RESPIRATORY ACIDOSIS EXPECTED PATIENT OUTCOMES include but are not limited to: 1 Will maintain airway patency and adequate breathing rate and rhythm will return of ABGs to patient’s normal level 2 Will be alert and oriented to time, place, and person, or to his or her normal baseline level of consciousness 3 Will cope with anxiety 4 Will exhibit effective coping and awareness of effective support systems 5 Will have secretions that are normal for self in amount and can be raised 6 Will maintain adequate rate and depth of respirations using pursed lip and other breathing techniques when necessary (as in the patient with COPD)
  • 150. NURSING MANAGEMENT OF PATIENT WITH RESPIRATORY ACIDOSIS INTERVENTIONS 1 Supporting effective gas exchange - Provide a position of comfort to allow ease of respiration - Obtain and monitor ABG results and VS. Refer accordingly - Provide and monitor supplemental oxygen as ordered - Turn the patient q2 and PRN - Provide pulmonary hygiene PRN - Maintain adequate hydration - Provide comfort measures such as mouth care - Assist with ADLs - Instruct patient regarding coughing and deep breathing and management of disease condition, especially COPD 2 Coping with disturbed thought processes - Do frequent neurologic assessments - Monitor and document person’s baseline LOC frequently
  • 151. NURSING MANAGEMENT OF PATIENT WITH RESPIRATORY ACIDOSIS - Reorient as necessary by providing calendars, clocks, etc. 3 Relieving anxiety - Provide a calm, relaxed environment - Give clear, concise explanations of treatment plans - Encourage expression of feelings - Provide support and information to patient and family - Teach relaxation techniques - Assist the patient to identify coping mechanisms to deal with anxiety and stress 4 Enhancing coping mechanisms - Provide support and information to family members about the patient’s ongoing condition - Reassure them that there is a physiologic cause for the patient’s behavior
  • 152. NURSING MANAGEMENT OF PATIENT WITH RESPIRATORY ACIDOSIS - Encourage questions and open communication 5 Promote airway clearance - Implement regular breathing and coughing exercises - Do suctioning as necessary - Maintain good hydration - Do chest physiotherapy as appropriate 6 Promoting an effective breathing pattern - Maintain alveolar ventilation - Teach the patient proper breathing techniques as well as panic control breathing
  • 153. NURSING MANAGEMENT OF PATIENT WITH RESPIRATORY ACIDOSIS EVALUATION. Achievement of outcomes is successful when the patient: 1a. Demonstrates improved ventilation and oxygenation 1b Has vital signs, ABGs, and cardiac rhythm within own normal range 2 Returns to baseline LOC 3 Reports reduced anxiety 4 Family uses adequate coping mechanisms 5 Is able to raise secretions on own 6 Demonstrate effective breathing techniques
  • 154. RESPIRATORY ALKALOSIS: CARBONIC ACID DEFICIT Anxiety, hysteria, fever, hypoxia, pain, pulmonary disorders, lesions affecting the respiratory center in the medulla, brain tumor, encephalitis, meningitis, hyperthyroidism, gram-negative sepsis Hyperventilation: Excessive pulmonary ventilation Decrease in hydrogen ion concentration RESPIRATORY ALKALOSIS
  • 155.
  • 156. NURSING MANAGEMENT OF RESPIRATORY ALKALOSIS ASSESSMENT * Health Hx: anxiety, shortness of breath, muscle cramps or weakness, palpitations, panic, dyspnea * Physical Examination: light-headedness, confusion as a result of cerebral hypoxia, hyperventilation, tachycardia or arrhythmia, muscle weakness, (+) Chvostek’s sign or Trousseau’s sign indicating a low ionized serum calcium level secondary to hyperventilation and alkalosis, hyperactive deep tendon reflexes, unsteady gait, muscle spasms to tetany, agitation, psychosis, seizures in extreme cases, decreased potassium levels NURSING DIAGNOSES include but are not limited to: Diagnostic Title Possible Etiologic Factors 1 Anxiety Stress, fear 2 Ineffective breathing pattern Hyperventilation, anxiety 3 Disturbed thought processes CNS excitability; irritability 4 Risk for injury Change in LOC, and potential for seizures
  • 157. NURSING MANAGEMENT OF RESPIRATORY ALKALOSIS EXPECTED PATIENT OUTCOMES include but are not limited to: 1 Will report decreased anxiety; verbalizes methods to cope with anxiety 2 Will return to normal respiratory rate and rhythm or at least decreased hyperventilation, with return to baseline ABGs 3 Will exhibit reorientation to person, place, and time as per patient’s baseline 4 Will be free from injury INTERVENTIONS 1 Allay anxiety - Give antianxiety medications as ordered - Have patient breath into a paper bag - Teach relaxation techniques when initial anxiety attack is over
  • 158. NURSING MANAGEMENT OF PATIENT WITH RESPIRATORY ACIDOSIS INTERVENTIONS 2 Promoting an Effective Breathing Pattern - Encourage the patient to slow his or her RR - Maintain a calm and comforting attitude - Position the patient to promote maximal ease of inspiration - Assist the patient with relaxation techniques 3 Coping with Disturbed Thought Processes - Do frequent reorientation - Encourage family to participate in patient’s care - Use simple, direct statements or directions - Allow the patient adequate time to respond 4 Preventing injuries - Perform neurologic assessment frequently and document - Institute safety and seizure precautions - Assess frequently for muscle strength and coordination
  • 159. NURSING MANAGEMENT OF PATIENT WITH RESPIRATORY ACIDOSIS EVALUATION. Achievement of outcomes is successful when the patient: 1 Reports reduction in anxiety levels 2a Demonstrates effective normal breathing patterns 2b Has ABG results within patient’s normal baseline 3 Returns to normal baseline LOC and orientation level 4 Remains free from injury; no seizure activity
  • 160. METABOLIC ACIDOSIS: BICARBONATE DEFICIT Increased acid production, uncontrolled diabetes mellitus, alcoholism, starvation, renal acidosis, lactic acidosis, increased acid ingestion, ethanol, salicylates, loss of bicarbonate, severe diarrhea, intestinal fistulas, adrenal insufficiency, hypoparathyroidism Excess organic acids are added to body fluids or bicarbonate is lost Decrease in bicarbonate concentration METABOLIC ACIDOSIS
  • 161.
  • 162. NURSING MANAGEMENT OF METABOLIC ACIDOSIS ASSESSMENT * Health Hx: anorexia, nausea, vomiting, abdominal pain, headache, thirst if the patient is dehydrated * Physical Examination: confusion, hyperventilation, warm, flushed skin, bradycardia and other dysrhythmias, decreasing LOC, nausea, vomiting, diarrhea, Kussmaul respirations, and acetone breath, especially if acidosis is due to ketoacidosis. Symptoms may progress to coma if untreated NURSING DIAGNOSES include but are not limited to: Diagnostic Title Possible Etiologic Factors 1 Disturbed thought processes Secondary to CNS depression 2 Decreased cardiac output Dysrhythmias 3 Risk for injury Secondary to altered mental state 4 Risk for imbalanced fluid Diarrhea, renal failure volume
  • 163. NURSING MANAGEMENT OF METABOLIC ACIDOSIS EXPECTED PATIENT OUTCOMES include but are not limited to: 1 Will return to usual baseline LOC 2 Will return to normal baseline parameters for vital signs with improved CO and decreased or resolved dysrhythmias 3 Will remain in a safe, secure environment without injury 4 Will maintain fluid and electrolyte balance and stable renal status INTERVENTIONS 1 Coping with disturbed thought processes - Monitor LOC and reorient as necessary - Monitor VS, esp. RRR, BP, and T - Monitor ABGs to assess the effects of treatment
  • 164. NURSING MANAGEMENT OF PATIENT WITH METABOLIC ACIDOSIS 2 Supporting cardiac output - Monitor VS, MIO, and fluid and electrolyte balance - Institute cardiac monitoring to evaluate cardiac status 3 Promoting safety - Provide a safe, secure and monitored environment - Institute safety precautions 4 Promoting return of fluid and electrolyte balance - Monitor MIO - Administer medications per medical order
  • 165. NURSING MANAGEMENT OF PATIENT WITH METABOLIC ACIDOSIS EVALUATION. Achievement of outcomes is successful when the patient: 1 Exhibits baseline-level consciousness and orientation 2 Returns to normal baseline parameters for vital signs and Cardiac Output with cardiac dysrhythmias resolved 3 Remains free from injury 4 Maintains fluid and electrolyte balance and stable renal function
  • 166. METABOLIC ALKALOSIS: BICARBONATE EXCESS Loss of stomach acid, gastric suctioning, persistent vomiting, excess alkali intake, intestinal fistulas, hypokalemia, Cushing’s syndrome or aldosteronism, potassium-diuretic therapy Excessive amounts of acid substance and hydrogen ions are lost from the body or large amounts of bicarbonate or lactate are added orally or IV Excess of base elements METABOLIC ALKALOSIS
  • 167.
  • 168. NURSING MANAGEMENT OF METABOLIC ALKALOSIS ASSESSMENT * Health Hx: Prolonged vomiting or nasogastric suctioning, frequent self-induced vomiting, muscle weakness, light- headedness, ingestion of large amounts of licorice or antacids, use of diuretics, muscle cramping, twitching, or tingling * Physical Examination: mental confusion, dizziness, changes in LOC, hyperreflexia, tetany, dysrhthmias, seizurees, respiratory failure, positive Chvostek’s or Trosseau’s sign if the patient has a low ionized serum calcium level, decreased hand grasps, generalized muscle weakness, decreased serum calcium or potassium level, impaired concentration, seizures, ECG changes consistent with hypokalemia NURSING DIAGNOSES include but are not limited to: Diagnostic Title Possible Etiologic Factors 1 Disturbed thought processes CNS excitation 2 Decreased cardiac output Dysrhythmias and electrolyte imbalances 3 Risk for injury Muscle weakness, tetany, confusion and possible seizures 4 Risk for imbalanced fluid volume Nasogastric drainage, diuretic therapy volume
  • 169. NURSING MANAGEMENT OF METABOLIC ALKALOSIS EXPECTED PATIENT OUTCOMES include but are not limited to: 1 Will return to usual baseline LOC and orientation 2 Will return to normal baseline parameters for vital signs with improved CO with resolution of electrolyte imbalances and decreased or resolved cardiac dysrhythmias 3 Will remain in a safe, secure environment without injury 4 Will maintain fluid and electrolyte balance INTERVENTIONS 1 Coping with disturbed thought processes - Monitor LOC and reorient as necessary - Monitor VS, esp. RRR, BP, and T - Monitor ABGs to assess the effects of treatment - Institute cardiac monitoring as ordered
  • 170. NURSING MANAGEMENT OF PATIENT WITH METABOLIC ALKALOSIS 2 Supporting cardiac output - Monitor VS, MIO, and fluid and electrolyte balance - Institute cardiac monitoring to evaluate cardiac status 3 Promoting safety - Provide a safe, secure and monitored environment - Institute safety precautions 4 Promoting return of fluid and electrolyte balance - Monitor MIO - Administer medications per medical order
  • 171. NURSING MANAGEMENT OF PATIENT WITH METABOLIC ALKALOSIS EVALUATION. Achievement of outcomes is successful when the patient: 1 Manifests mental status has returned to baseline 2 Is free from cardiac dysrhythmias 3 Remains free from injury 4 Maintains fluid balance at baseline level
  • 172. CRITICAL THINKING EXERCISES A 32-year-old administrative assistant comes to the urgent care center with a 72-hour history of vomiting secondary to influenza. She is lethargic and states, “My muscles are twitching.” Her RR is 18/min and HR is 110 bpm, T=100.4F. Her blood pressure is 110/68 which she states “is about normal for me.” Her ABG values are as follows: pH: 7.57 PaO2: 92 PaCO2: 41 HCO3: 36 Describe her acid-base status, probable cause for the imbalance and treatment